Section 6 General Internal Medicine
392
SECTION 6 General Internal Medicine
393
CHAPTER 19
394
Delirium
395 Case: A 27-year-old man with flushed skin A 27-year-old man is brought to the emergency department by law
enforcement after he was found undressed and wandering the streets. restrained. The patient’s wife is at his side and reports that he was prescription medications but has been consuming escalating quantities According to his wife, he has taken “handfuls” of pills daily over the without any clear focus of attention, frequently picking atlhisrbodypre Bowel sounds are Decreased..A photograph of the patient in a well-lit He became agitated when confronted and had to be forcefully previously healthy except for seasonal allergies. He takes no of over-the-counter allergy medication over the past few weeks. past few days. Temperature is 38.6°C, and heart rate is 125 beats per minute. The is generalized skin flushing. The patient is looking about the oom and nearby bedsheets. He is disoriented and unable to fol ow 2-ste commands. He is unable to state the days of the week backward. room is shown in Figure 19-1
FIGURE 19-1 (Courtesy of Ross Passo, MD.)
What is the most likely diagnosis in this patient?
What is delirium?
How can attention be evaluated on examination?
In addition to cognitive disturbances, what other clinical features are common in patients with delirium? What is altered mental status?
What is encephalitis?
What is limbic encephalitis?
Delirium is a clinical diagnosis, defined as an acute disturbance in attention and awareness (lasting hours to days), accompanied by an additional disturbance in cognition (ie, memory, orientation, language, visuospatial ability, or perception). There must be evidence that the disturbance is caused by a medical condition, withdrawal or intoxication from medication or illicit substances, is not better explained by a preexistent neurocognitive disorder, and is not occurring in the context of a severely reduced level of arousal (eg, coma).1,2 Inattention is the neurocognitive hallmark of delirium. It can be observed simply by taking a history from a patient. It may manifest as tangential speech, fragmentary flow of ideas, or inability to follow complex commands. Asking the patient to repeat successively longer random strings of digits is one way to actively assess attention. The average adult can repeat a string of 5 to 7 digits without failing; inability to achieve this target is indicative of inattention. More formal neuropsychological testing may be helpful.3 Patients with delirium often experience fluctuating symptoms over the course of the day and may even appear lucid at times. Disturbances in perception (eg, visual hallucinations), the sleep-wake cycle, psychomotor activity (ie, hypoactivity or hyperactivity), emotional control, and behavior regulation are common.4
Unlike delirium, the term “altered mental status” is nonspecific and poorly defined. It is generally used to indicate a change in cognitive function or level of consciousness. Other nonspecific and poorly defined terms often used interchangeably with delirium include encephalopathy and acute confusional state.5 Encephalitis refers to inflammation of the brain that may result in delirium and other characteristic clinical manifestations depending on the affected location (eg, limbic system, brainstem). Encephalitis is most often infectious in nature but can also occur as a result of noninfectious inflammatory conditions (eg, autoimmune encephalitis).6 Limbic encephalitis refers to a particular clinical syndrome that occurs as a result of inflammation of the limbic system. It is most often caused by autoimmune or paraneoplastic syndromes. Characteristic clinical features
396
In a patient with psychosis, when should causes other than delirium be suspected? How common is delirium in hospitalized patients? What predisposing factors increase the risk of delirium in hospitalized patients? What is the role of neuroimaging in patients with delirium?
What is the role of electroencephalography (EEG) in patients with delirium?
What is the role of lumbar puncture in patients with delirium? What are the principles of managing delirium?
include rapid-onset confusion, working memory deficits, mood changes, and seizure.6 Causes of psychosis other than delirium (eg, drug-induced psychosis, schizophrenia, bipolar mania) should be considered when patients experience auditory hallucinations, delusions, and disorganized thoughts and behaviors, but relatively preserved alertness and memory.7 Delirium is a common disorder in the hospital setting, developing in up to one-third of patients. It is particularly common in patients ≥65 years of age, developing in up to one-half of those hospitalized.4,8 Predisposing factors that increase risk of delirium in hospitalized patients include older age, dementia, cognitive impairment, depression, alcohol abuse, functional impairment, sensory impairment (eg, visual, auditory), history of delirium, and history of stroke.4 Neuroimaging (eg, computed tomography and magnetic resonance imaging) is normal in the vast majority of patients with delirium, limiting its diagnostic yield. However, neuroimaging should be obtained in select patients with delirium, such as those with focal neurologic findings, to evaluate for etiologies such as stroke, intracranial hemorrhage (ICH), and brain abscess.4 Delirium is associated with the characteristic EEG pattern of diffuse slowing, with poor organization of background rhythm; certain etiologies of delirium may have other distinctive features (eg, herpes simplex encephalitis is associated with lateralized periodic discharges). However, EEG is not routinely obtained in the evaluation of delirium. In select cases when the diagnosis is uncertain, EEG can be useful in differentiating organic disease from functional or psychiatric disorders, and in identifying occult seizures.4 Evaluation of cerebrospinal fluid (CSF) should be considered in select patients with delirium who are suspected of having conditions such as meningitis, encephalitis, or subarachnoid hemorrhage.4 Other than addressing the underlying cause(s), management of delirium includes minimizing centrally acting medications (eg, narcotics and anticholinergics), maximizing presence of and interaction with friends and family, decreasing sleep-wake disturbance, adequately treating pain, and maintaining a quiet environment.4
When are pharmacologic agents useful in the management of delirium? The causes of delirium can be separated into which general categories?
Medications, such as antipsychotic agents, should be considered in delirious patients with severe agitation who exhibit behavior that may be harmful to themselves or others, and in those with distressing psychotic symptoms.4 The causes of delirium can be separated into the following categories: neurologic, toxic, metabolic, infectious, and other.
397
Neurologic Causes of Delirium
What are the neurologic causes of delirium?
A 78-year-old man Stroke. with a history of atrial fibrillation presents with confusion and left- sided hemiparesis. EEG can confirm Seizure. the diagnosis. A high school Concussion. running back presents to the student health center with persistent disorientation after sustaining a hard hit to the head during a recent football game. An acute Intracranial hemorrhage. complication of traumatic brain injury (TBI) that can be diagnosed with neuroimaging or evaluation of CSF. Astrocytoma, Brain tumor. oligodendroglioma, and ependymoma. Enlarged ventricles Hydrocephalus. on neuroimaging. Blood vessel wall Central nervous system (CNS) vasculitis, including primary angiitis of the central nervous system (PACNS), inflammation. systemic vasculitis (eg, Behçet’s disease), and vasculitis associated with systemic disease (eg, systemic lupus erythematosus). Noninfectious Immune-mediated encephalitis, including paraneoplastic encephalitis, autoimmune encephalitis, Hashimoto’s causes of encephalitis, acute disseminated encephalomyelitis, Bickerstaff’s brainstem encephalitis, and IgG4-related disease. encephalitis.
What are the causes of cerebral hypoxia? What is the significance of delirium in patients with acute stroke? How long does postseizure delirium typically last? What are the characteristics of delirium in patients with traumatic brain injury? Which type of intracranial hemorrhage tends to affect elderly
Causes of cerebral hypoxia include stroke, hypoxemia from any cause (eg, pneumonia), systemic hypotension from any cause (eg, myocardial infarction), severe anemia from any cause (eg, autoimmune hemolytic anemia), and toxins (eg, carbon monoxide). Delirium occurs in up to one-third of patients admitted to the hospital with acute stroke. It is associated with Increased duration of hospitalization, poorer functional outcomes, and Increased mortality. The 12-month mortality is 5 times higher in patients with poststroke delirium than in stroke patients without delirium.9
Delirium can be challenging to distinguish from seizure; particularly focal seizure with impairment of consciousness, and nonconvulsive status epilepticus. Many of the causes and clinical features overlap, however delirium tends to be more gradual in onset. EEG can often be helpful in distinguishing the 2 conditions. True postseizure delirium usually resolves within hours of the event but can last for days in some cases.7 Delirium is common in patients hospitalized for TBI, usually developing within the first day of the trauma. Although most manifestations resolve within a few days, agitation and disruption of the sleep-wake cycle tend to last longer.10
Chronic subdural hematoma is a common cause of cognitive impairment in the elderly. It describes the presence of blood in the subdural space, usually caused by minor trauma. Gait disturbance, limb weakness, headache, and delirium are common features, typically progressing over a period of days to weeks. Noncontrast CT imaging of the head is diagnostic. Bleeding diatheses should be corrected when possible; most symptomatic patients require
398
patients, often presenting weeks after minor trauma? Are neurocognitive symptoms more typical of primary or metastatic brain tumors? What is the classic triad of normal pressure hydrocephalus (NPH)?
surgical management.11
Metastatic tumors, which account for up to one-quarter of all brain tumors, tend to involve more areas of the brain compared with primary tumors, which increases the likelihood of neurocognitive symptoms.12
NPH is a disease of older adults characterized by the triad of gait disturbance, urinary incontinence, and cognitive deficits. The early neurocognitive symptoms of NPH include psychomotor slowing; and impaired attention, executive, and visuospatial function. Neuroimaging reveals enlarged cerebral ventricles (Figure 19-2). Timely diagnosis and treatment with CSF shunting procedures (eg, ventriculoperitoneal shunt placement) can lead to resolution of symptoms. Although it is usually categorized as a reversible dementia, NPH should be considered in the differential diagnosis of delirium.13
FIGURE 19-2 Computed tomography imaging of the brain reveals bilateral ventricular enlargement in a patient with
How common is delirium in patients with primary angiitis of the central nervous system? What is autoimmune encephalitis?
What is paraneoplastic encephalitis?
normal pressure hydrocephalus. (From Garcia MJ. Noninvasive Cardiovascular Imaging: A Multimodality Approach. Philadelphia, PA: Lippincott Williams & Wilkins; 2010.)
Delirium occurs in one-half of patients with PACNS at the time of presentation. Other frequent manifestations include headache and focal neurologic findings. CSF analysis reveals pleocytosis or elevated protein in most patients. Neuroimaging, including cerebral angiography, and brain biopsy are often needed to make the diagnosis.14
Autoimmune encephalitis refers to a group of disorders that cause encephalitis, often in association with antibodies to neuronal cell-surface or synaptic proteins. Anti-N-methyl-d-aspartate (anti-NMDA) receptor encephalitis is the most well-described type of autoimmune encephalitis. It predominantly affects women and patients under 45 years of age. Presenting symptoms include abnormal behavior, psychosis, speech dysfunction, dyskinesia, memory deficits, autonomic instability, Decreased level of consciousness, and seizure. CSF is abnormal in most patients and will often demonstrate a moderate lymphocytic pleocytosis with normal or mildly Increased protein concentration. CSF-specific oligoclonal bands are present in most patients. The detection of anti-NMDA receptor antibodies, which are present in the vast majority of affected patients, confirms the diagnosis.6,15 Paraneoplastic encephalitis refers to encephalitis caused by a distant underlying malignancy. It typically manifests as limbic or brainstem encephalitis. It can be associated with a variety of malignancies, but small cell lung cancer is responsible for most cases. The onset of neurologic symptoms precedes the diagnosis of cancer in most patients. Characteristic antibodies (eg, anti-Hu) can be found in the serum and CSF of affected patients.16
399
Toxic Causes of Delirium
What are the toxic causes of delirium?
One of the most common causes of delirium, particularly in the elderly. This substance can cause delirium through a variety of mechanisms, including intoxication, withdrawal, and associated vitamin deficiencies. After becoming confused at a party, a 22-year-old woman is taken to the emergency department. Said Voltaire, “A dish of mushrooms changed the destiny of Europe.”
Which medications can cause delirium?
What is delirium tremens?
Which recreational drugs can cause delirium? Which poison should be suspected in a patient with delirium and cherry-red lips?
Medication.
Alcohol.
Recreational drug use.
Amanita phalloides (ie, the “death cap” mushroom).17
Numerous medications can cause delirium. The most frequent culprits include sedative hypnotics (eg, benzodiazepines), analgesics (eg, narcotics), and anticholinergics (eg, antihistamines, which are available over-the-counter). Other delirium-producing medications include anticonvulsants (eg, barbiturates), muscle relaxants, glucocorticoids, tricyclic antidepressants, antiemetics (eg, scopolamine), digoxin, lithium, and fluoroquinolones. Keys to identifying and treating medication-induced delirium include a thorough review of the medication list (including over-the-counter and herbal substances), reviewing for possible adverse interactions between medications (eg, serotonin syndrome), and recognizing the potential for altered pharmacokinetics (eg, renal or hepatic impairment). It is important to be aware that delirium may be part of a withdrawal syndrome, particularly involving sedative, hypnotic, and anxiolytic substances.18 Delirium can be caused by both alcohol intoxication and withdrawal. Delirium tremens is a life-threatening manifestation of alcohol withdrawal, characterized by rapid onset disturbances in attention and cognition, sometimes with hallucinations and extreme autonomic hyperactivity (eg, fever, tachycardia, hypertension). Onset is typically 72 hours after the appearance of alcohol withdrawal symptoms and lasts for several days. It is most often treated with benzodiazepines.19 Virtually any recreational drug can cause delirium. Common culprits include amphetamines (eg, methamphetamine and 3,4-methylenedioxymethamphetamine [MDMA]), cocaine, heroin, γ-hydroxybutyric acid (GHB), benzodiazepines, ketamine, phencyclidine (PCP), lysergic acid diethylamide (LSD), and psilocybin (mushrooms). It is important to be aware that delirium may be part of a withdrawal syndrome, particularly involving sedative, hypnotic, and anxiolytic substances.3 The presence of cherry-red lips can be a clue to the diagnosis of carbon monoxide poisoning. Other poisons that can cause delirium include methanol, ethylene glycol, pesticides (eg, organophosphates), cyanide, mushrooms (particularly Amanita phalloides), and Datura stramonium (jimson weed).3
400
401
Metabolic Causes of Delirium
What are the metabolic causes of delirium?
Associated with elevated blood ammonia levels. Auscultation of the heart reveals the presence of a scratchy sound with 3 components. A 22-year-old woman with type 1 diabetes mellitus (hemoglobin A1C of 5.8%) presents with confusion and somnolence. A 66-year-old man with type 2 diabetes mellitus (hemoglobin A1C of 13.8%) presents with confusion and dehydration. A 28-year-old woman with anorexia nervosa is admitted to the hospital and develops delirium 2 days after being started on parenteral nutrition. A 60-year-old man with chronic alcohol use is admitted with confusion and ataxia, which acutely worsen after he is given a meal. A 66-year-old man with severe chronic obstructive pulmonary disease is admitted to the hospital with confusion, obtundation, and poor air movement. Endocrinopathies. Osborn waves on the electrocardiogram (Figure 19-3).
Hepatic encephalopathy.
Uremia.
Hypoglycemia.
Hyperosmolar hyperglycemic state.
Electrolyte disturbances caused by refeeding syndrome.
Wernicke’s encephalopathy (thiamine deficiency).
Hypercarbia.
Thyroid disease (both hyperthyroidism and hypothyroidism) and Cushing’s syndrome. Hypothermia.
FIGURE 19-3 An electrocardiogram demonstrating prominent Osborn waves (arrow) in a patient with hypothermia.
A 32-year-old woman Hyperthermia caused by neuroleptic malignant syndrome. with schizophrenia presents with fever, muscle rigidity, autonomic lability, and delirium after a recent increase in haloperidol dose.
402
What is considered First-line pharmacologic treatment for hepatic encephalopathy is a nonabsorbable disaccharide (eg, lactulose). It first-line should be dosed to achieve 2 to 4 semisoft bowel movements per day. Colonic bacteria metabolize these pharmacologic disaccharides, which creates an acidic environment that is hostile to the intestinal bacteria that produce ammonia. treatment for hepatic This acidic environment also facilitates the conversion of ammonia to a nonabsorbable form (NH →NH ), 3 4 + encephalopathy? essentially trapping it in the colon for excretion. These effects combine to decrease the amount of ammonia entering the circulation.20 In addition to uremia, Decreased renal clearance of medications and metabolites can lead to toxicity at doses that were previously what is another major therapeutic. A thorough review of the medication list is important in any patient with reduced renal function; mechanism by which medications should be removed or dosed accordingly. end-stage renal disease contributes to the development and persistence of delirium? What is the threshold Normal metabolism in the brain relies on an adequate concentration of glucose in blood. In healthy research blood glucose level at subjects with insulin-induced hypoglycemia, cognitive impairment begins to occur when blood glucose levels which neurocognitive drop to about 50 mg/dL. Neuroglycopenic manifestations include confusion, tiredness, speaking difficulties, impairment headache, inability to concentrate, behavioral changes, seizure, and coma. The vast majority of cases of develops? hypoglycemia occur as an unintentional result of the pharmacologic treatment for diabetes mellitus. Other causes of hypoglycemia include medications (eg, β-blockers), toxins (eg, ethanol), adrenal insufficiency, intense exercise, insulinoma, critical illness (eg, liver failure), and surreptitious insulin use.21 What are the Plasma hypertonicity leads to fluid shift from the intracellular fluid compartment to the extracellular fluid principal mechanisms compartment, with resultant neuronal dehydration and dysfunction. Osmotic diuresis may also lead to electrolyte of neurocognitive disturbances that contribute to neurocognitive dysfunction.22 dysfunction in patients with hyperosmolality? Which electrolyte Electrolyte disturbances most frequently associated with delirium include hyponatremia, hypernatremia, disturbances can hypercalcemia, hypocalcemia, and hypomagnesemia. The likelihood of neurocognitive impairment from an cause delirium? electrolyte disturbance depends on the severity of the disturbance and its rate of development.3 Which vitamin Delirium can be caused by deficiencies in thiamine, vitamin B12, folate, and niacin.3 deficiencies can cause delirium? What are the Hypercarbia causes delirium through a variety of mechanisms. First, it increases intracranial pressure via its mechanisms of vasodilatory effect on cerebral vasculature. Second, it leads to CSF and brain tissue acidosis, which further delirium in patients increases cerebral blood flow and intracranial pressure. Finally, hypercarbia causes systemic hypoxemia, which with hypercarbia? can lead to cerebral hypoxia. Mechanical ventilation is the treatment of choice for hypercarbia complicated by delirium.23 What are the Neurocognitive manifestations occur frequently in patients with hypothyroidism and are sometimes the earliest neurocognitive and most prominent expressions of the condition. Early symptoms include inattention, memory deficits, and features of mental slowness; later symptoms include delusions and hallucinations. Early recognition and treatment may hypothyroidism? reverse the cognitive effects of hypothyroidism; however, deficits may persist in some cases.24 How common is Neurocognitive impairment occurs in most patients with Cushing’s syndrome, including deficits in memory, neurocognitive attention, and concentration. Treatment for Cushing’s syndrome ameliorates these symptoms in most patients; impairment in however, symptoms may be irreversible or slow to improve, particularly in the elderly.25 patients with Cushing’s syndrome? What are the Hypothermia (core temperature <35°C) can occur when the thermoregulatory actions of the body are either neurocognitive overwhelmed by exposure to a cold environment or impaired as a result of a variety of medical conditions, such as manifestations of stroke, alcohol ingestion, hypothyroidism, or sepsis. Older patients are particularly susceptible to hypothermia. hypothermia? Confusion and memory impairment are among the earliest neurocognitive manifestations. With progression, patients develop apathy, impaired judgment (eg, paradoxical undressing), and dysarthria. If hypothermia progresses, consciousness is ultimately lost.26 What are the Hyperthermia can be caused by infectious and noninfectious etiologies, such as heat stroke and medications. neurocognitive Neurocognitive manifestations include impairment in attention, memory, reasoning, problem solving, manifestations of comprehension, and consciousness. These deficits resolve in most patients. However, some are left with hyperthermia? permanent changes in attention, memory, or personality.27
403
Infectious Causes of Delirium
What are the infectious causes of delirium?
Delirium may be the only clinical manifestation of this common infection, particularly in the elderly. Purulent cough and fever. Any infection with signs of systemic toxicity. Neck stiffness, photophobia, and headache. Most commonly viral in nature, this CNS infection is usually associated with pleocytosis. This CNS infection may present with focal neurologic findings.
Which class of antibiotics, commonly used to treat urinary tract infection, should be avoided in older patients who are at risk for delirium? How is the prognosis of elderly patients with pneumonia affected by the presence of delirium? What is sepsis-induced delirium?
Is meningitis or encephalitis more likely to present with neurocognitive dysfunction early in the course of illness? What are the most common symptoms reported by patients with brain abscess?
Urinary tract infection.
Pneumonia. Sepsis. Meningitis. Encephalitis.
Brain abscess.
Fluoroquinolones can cause delirium and should generally be avoided in patients with risk factors, including advanced age.18
The presence of delirium in elderly patients with pneumonia increases the risk of in-hospital mortality by a factor of 6. This is approximately equivalent to the Increased risk associated with the presence of underlying chronic obstructive pulmonary disease in patients with pneumonia.28 Delirium due to sepsis is caused by the systemic inflammatory response to infection originating outside of the central nervous system. It is present in up to one-quarter of septic patients and may even precede the typical features of sepsis. The presence of delirium in septic patients doubles the rate of mortality.29 Compared with meningitis, encephalitis is more likely to be associated with neurocognitive dysfunction early in the course of illness. Viruses are responsible for most cases of encephalitis but must be distinguished from immune-mediated processes (eg, autoimmune encephalitis). Neuroimaging and CSF evaluation are key investigations in patients with encephalitis. Importantly, normal CSF findings (including the absence of pleocytosis) are present in up to 10% of patients with viral encephalitis.30 Headache is the most frequent symptom reported by patients with brain abscess. Fever, delirium, focal neurologic findings, and nausea and vomiting are also common manifestations. The classic triad of fever, headache, and focal neurologic findings is present in less than one-half of patients. Neuroimaging is key to the diagnosis.31
404
Other Causes of Delirium
What are the other causes of delirium?
A common occurrence in hospitalized patients owing to a noisy environment, constant interruptions, artificial light, pain, and underlying illness. Acute headache, confusion, and papilledema in a patient with an S4 gallop and the presence of left ventricular hypertrophy on electrocardiogram. Headache, vision changes, delirium, and the presence of cerebral edema predominantly involving the white matter of the posterior regions of the brain.
Which pharmacologic agent may be helpful in preventing insomnia and delirium in hospitalized patients? What is hypertensive encephalopathy?
What is posterior reversible encephalopathy syndrome?
Insomnia.
Hypertensive encephalopathy.
Posterior reversible encephalopathy syndrome (PRES).
Melatonin or melatonin agonists (eg, ramelteon) may be useful in the prevention and treatment of delirium in hospitalized patients. Ramelteon is well-tolerated and, unlike other pharmacologic sleep aids, is not associated with cognitive impairment, withdrawal symptoms, rebound insomnia, or potential abuse.32
Hypertensive encephalopathy occurs when systemic hypertension overwhelms the autoregulatory capacity of the cerebral vasculature, resulting in hyperperfusion, cerebral edema, and delirium. Common manifestations include lethargy, confusion, headache, visual disturbance, and seizure. Neuroimaging may reveal the presence of cerebral edema predominantly involving the white matter of the parietal and occipital regions, a finding consistent with PRES.33 PRES describes the development of neurocognitive dysfunction in association with cerebral edema that develops as a result of endothelial dysfunction, typically involving the posterior brain regions (Figure 19-4). It most often arises when there are pronounced fluctuations in systemic blood pressure, particularly abrupt and severe elevations. Other associated conditions include renal failure, use of cytotoxic medications, autoimmune disorders (eg, systemic lupus erythematosus), and preeclampsia or eclampsia. Treatment is aimed at the precipitating condition. Most patients fully recover within 1 week.34
FIGURE 19-4 Brain MRI shows symmetric patchy areas of cortical and subcortical abnormal signal in the parieto- occipital lobes corresponding to vasogenic edema (arrows), findings characteristic of PRES. (From Sanelli PC, Schaefer P, Loevner LA. Neuroimaging: The Essentials. Philadelphia, PA: Wolters Kluwer Health; 2016.)
405
406 Case Summary quantitiesfof over-the-counter allergy medication and istfound to.have What is the most likely diagnosis in this patient? Anticholinergic syndrome. A 27-year-old man presents with agitation after inges ing large delirium, ever, tachycardia, and abnormal skin and eye findings
407
Bonus Questions
What is anticholinergic Anticholinergic syndrome is a toxidrome resulting from the inhibition of neurotransmission at muscarinic acetylcholine receptors sites, which are located syndrome? both centrally (ie, within the CNS) and peripherally (eg, within the heart, Gastrointestinal tract, and sweat glands). 35,36 What are the clinical Anticholinergic syndrome includes manifestations related to both central and peripheral muscarinic receptor blockade. Inhibition of the receptors located in manifestations of the CNS can result in delirium, psychosis, hallucinations, seizure, and coma. Inhibition of the peripheral receptors can result in hyperthermia, tachycardia, anticholinergic syndrome? anhidrosis, dry mucous membranes, skin flushing (vasodilation), mydriasis, Decreased bowel sounds, and urinary retention.35,36 What findings are present Mydriasis and skin flushing can be appreciated in the photograph of the patient in this case (see Figure 19-1), both of which are signs of anticholinergic in the photograph of the toxicity. patient in this case? Which classes of Classes of medications with anticholinergic properties include antihistamines, tricyclic antidepressants, antipsychotics, antidiarrheals, antiemetics, medications arelcapable of antiparkinsonian agents, antispasmodics, bronchodilators, mydriatics, and skeletal muscle relaxants. Poisons such as Datura stramonium (jimson weed) are syndrome? causing anticho inergic also capable of causing anticholinergic syndrome. 35,36 How is the diagnosis of Anticholinergic syndrome s a clinical diagnosis. A compatible history, includ ng exposure to medications or other substances with anticholinergic anticholinergic syndrome properties, must be soughtifrom family or friends. Laboratory investigation isioften fruitless, as many anticholinergic agents are not detected on toxicology made? screens and results are slow to return. What is the manag Supportive management is the cornerstone of treatment for anticholinergic syndrome. Depending on the degree of agitation, sedatives such as 36 of anticholinergic ement benzodiazepines may also be needed. Physostigmine prevents the degradation oflacetylcholine and can be used as an antidote; however, it can cause serious syndrome? side effects, including seizure and cardiac dysrhythmia, so its use must be careful y considered. 36
408 Key Points
and awareness, along withtadditional disturbances in cognition. g involving perception, the sleep-wake cycle, psychomotor activity, Delirium is a clinical diagnosis caused by one or more underlyin medical conditions, charac erized by a disturbance in attention Additional features of delirium include fluctuating disturbances emotional control, and behavior regulation. Risk factors for delirium include older age, dementia, cognitive
impairment, history of delirium, functional impairment, sensory impairment, history of stroke, alcohol abuse, and depression. The causes of delirium can be separated into the following
categories: neurologic, toxic, metabolic, infectious, and other.ining Treatment for delirium hinges on.the identification and reversal of History and physical examination are fundamental to determ the cause(s) of delirium. Neuroimaging, lumbar puncture, and EEG can be helpful in some cases the underlying medical condition(s). Other basic tenets of treating delirium include minimizing
centrally acting medications (eg, narcotics), maximizing presence pain, and maintaining a quiet environment. of family, avoiding sleep-wake disturbances, adequately treating Pharmacologic treatment (eg, antipsychotic agents) should be considered in patients with severe agitation who exhibit behavior distressing psychotic symptoms. that may be harmful to themselves or others, and in those with
409
References 1. American Psychiatric Association. Diagnostic and Statistical Manual of Mental Disorders. 5th ed. Washington, DC: American Psychiatric Publishing; 2013.
-
European Delirium Association,iAmerican Delirium Society. The DSM-5 criteria, level 3. Longo DL, Fauci AS, Kasper DL, Hauser SL, Jameson JL, Loscalzo J, eds. Harrison’s of arousal and delirium diagnos s: inclusiveness is safer. BMC Med. 2014;12:141. Principles of Internal Medicine. 18th ed. New York, NY: McGraw-Hill; 2012.
-
Inouye SK, Westendorp RG, Saczynski JS. Delirium in elderly people. Lancet. 2014;383(9920):911-922.
-
Douglas VC, Josephson SA. Altered mental status. Continuum. 2011;17(5 Neurologic Consultation in the Hospital):967-983.
-
Graus F,lTitulaer MJ, Balu R, et al. A clinical approach to diagnosis of autoimmune encepha itis. Lancet Neurol. 2016;15(4):391-404.
- Kaplan PW. Delirium and epilepsy. Dialogues Clin Neurosci. 2003;5(2):187-200.
-
Siddiqi N, House AO, Holmes JD. Occurrence and outcome of delirium in medical in- patients: a systematic literature review. Age Ageing. 2006;35(4):350-364.
-
Shi Q, Presutti R, Selchen D, Saposnik G. Delirium in acute stroke: a systematic review and meta-analysis. Stroke. 2012;43(3):645-649.
-
Maneewong J, Maneeton B, Maneeton N, et al. Delirium after a traumatic brain injury: predictors and symptom patterns. Neuropsychiatr Dis Treat. 2017;13:459-465. . Kolias AG, Chari A, Santarius T, Hutchinson PJ. Chronic subdural haematoma:
11modern management and emerging therapies. Nat Rev Neurol. 2014;10(10):570-578. 12. Madhusoodanan S, Ting MB, Farah T, Ugur U. Psychiatric aspects of brain tumors: a review. World J Psychiatry. 2015;5(3):273-285.
-
Nassar BR, Lippa CF. Idiopathic normal pressure hydrocephalus: a review for general practitioners. Gerontol Geriatr Med. 2016;2:2333721416643702.
-
Salvarani C, Brown RD Jr, Calamia KT, et al. Primary central nervous system vasculitis: analysis of 101 patients. Ann Neurol. 2007;62(5):442-451. . Dalmau J, Lancaster E, Martinez-Hernandez E, Rosenfeld MR, Balice-Gordon R
15Clinical experience and laboratory investigations in patients with anti-NMDAR . encephalitis. Lancet Neurol. 2011;10(1):63-74. . Sillevis Smitt P, Grefkens J, de Leeuw B, et al. Survival and outcome in 73 anti-H
16positive patients with paraneoplastic encephalomyelitis/sensory neuronopathy. J u Neurol. 2002;249(6):745-753.
Wasson RG. The Death of Claudius, or Mushrooms for Murderers. Botanical Museum Leaflets, Harvard University. 1972;23(3):101-128.
-
Alagiakrishnan K, Wiens CA. An approach to drug induced delirium in the elderly. Postgrad Med J. 2004;80(945):388-393.
-
Schuckit MA. Recognition and management of withdrawal delirium (delirium tremens). N Engl J Med. 2014;371(22):2109-2113.
-
Sharma P, Sharma BC. Disaccharides in the treatment of hepatic encephalopathy. Metab Brain Dis. 2013;28(2):313-320.
-
Service FJ. Hypoglycemic disorders. N Engl J Med. 1995;332(17):1144-1152.
- Maccario M. Neurological dysfunction associated with nonketotic hyperglycemia. Arch Neurol. 1968;19(5):525-534.
410
- Scala R. Hypercapnic encephalopathy syndrome: a new frontier for non-invasive ventilation? Respir Med. 2011;105(8):1109-1117. . Heinrich TW, Grahm G. Hypothyroidism presenting as psychosis: myxedem
24madness revisited. Prim Care Companion J Clin Psychiatry. 2003;5(6):260-266.a 25glucocorticoid administration. Endocrinol Metab Clin North Am. 2013;42(3):477-488. . Starkman MN. Neuropsychiatric findings in Cushing syndrome and exogenous 26. Mallet ML. Pathophysiology of accidental hypothermia. QJM. 2002;95(12):775-785. 27. Walter EJ, Carraretto M. The neurological and cognitive consequences of hyperthermia. Crit Care. 2016;20(1):199. . Pieralli F, Vannucchi V, Mancini A, et al. Delirium is a predictor of in-hospital
28mortality in elderly patients with community acquired pneumonia. Intern Emerg Med. 2014;9(2):195-200.
- Zampieri FG, Park M, Machado FS, Azevedo LC. Sepsis-associated encephalopathy: not just delirium. Clinics. 2011;66(10):1825-1831. . Tunkel AR, Glaser CA, Bloch KC, et al. The management of encephal tis: clinical
30practice guidelines by the Infectious Diseases Society of America. CliniInfect Dis. 2008;47(3):303-327.
- Patel K, Clifford DB. Bacterial brain abscess. Neurohospitalist. 2014;4(4):196-204.
-
Chakraborti D, Tampi DJ, Tampi RR. Melatonin and melatonin agonist for delirium in the elderly patients. Am J Alzheimers Dis Other Demen. 2015;30(2):119-129.
-
Vaughan CJ, Delanty N. Hypertensive emergencies. Lancet. 2000;356(9227):411-417. 34. Fugate JE, RabinsteintAA. Posterior reversible encephalopathy syndrome: clinical and radiological manifesta ions, pathophysiology, and outstanding questions. Lancet Neurol. 2015;14(9):914-925.
-
Corallo CE, Whitfield A, Wu A. Anticholinergic syndrome following an unintentional overdose of scopolamine. Ther Clin Risk Manag. 2009;5(5):719-723.
-
Dart RC, ed..Medical Toxicology. 3rd ed. Philadelphia, PA: Lippincott Williams & Wilkins; 2004
411
CHAPTER 20
412
Dyspnea
413 Case: A 71-year-old woman with facial edema pulmonary disease (COPD) is admitted to the hospital withe A 71-year-old woman with a history of chronic obstructiv progressive shortness of breath. She is a former smoker with a 60-
pack-year history. Recent spirometry shows forced expiratory volume city blocks before needing to stop to catch her breath. Over the past capacity, now becoming short of breath with minimal exertion. Over swelling of the arms and face, and unintentional weight loss of 1 in 1 second (FEV ) of 55% of predicted. At baseline, she can walk 4 to 5 few weeks she has noticed a marked decrease in her functional the same period of time she has developed nonproductive cough, 15 pounds. peHeart rate is 106 beats per minute, and respiratory rate is 28 breaths 2 r minute. Jugular venous pressure (JVP) is 14 cm H O, and the external jugular vein is engorged. There is edema of the face, neck, and which is followed 30 seconds later by the onset of facial plethora. , upper extremities. Photographs of the patient are shown in Figure 20- 1A and B. The patient is asked to elevate her arms above her head A chest radiograph shows a left apical mass.
FIGURE 20-1
What is the most likely diagnosis in this patient?
What is dyspnea?
How is breathing regulated?
Dyspnea is the subjective experience of breathing discomfort or breathlessness. Patients perceive dyspnea in a variety of ways, including Increased work or effort of breathing, tightness, and air hunger (ie, unsatisfying inspiration), often occurring together. Dyspnea on exertion is considered abnormal when it occurs at a level of activity that is usually well tolerated.1,2 Respiratory drive is controlled by the respiratory centers in the brainstem, which are regulated by various central and peripheral sensory inputs. Examples of sensory inputs include central chemoreceptors, peripheral chemoreceptors, pulmonary stretch receptors, pulmonary C-fibers, and peripheral proprioceptors and metaboreceptors (Figure 20-2). Afferent information from the respiratory centers produces the sensation of dyspnea when an increase in respiratory drive is not satisfied (afferent/efferent mismatch).1,3,4
414
FIGURE 20-2 Regulation of respiration. Respiratory drive is dependent on information from various sensory inputs. + indicates Increased respiration; − indicates Decreased respiration; ± means that response may vary with circumstances. (From Taylor JJ. Memmler’s Structure and Function of the Human Body. 10th ed. Philadelphia, PA: Wolters Kluwer Health; 2013.
What are the Dyspnea is strictly a sensation that can only be experienced and reported by a patient, whereas tachypnea and differences hyperventilation are objective and measurable, and may or may not be associated with dyspnea. Tachypnea describes between dyspnea, an increase in respiratory rate above normal. Hyperventilation describes an increase in minute ventilation relative to tachypnea, and metabolic demand.2 hyperventilation? Which 2 organ Most causes of dyspnea are related to the heart or the lungs. systems are responsible for most causes of dyspnea?
What are the roles Aerobic respiration requires the exchange of oxygen and other nutrients for carbon dioxide and other waste products. of the heart and The lungs are the sites of oxygen and carbon dioxide exchange, whereas the heart provides the means of delivery. lungs in cellular Dyspnea is most often the result of failure of one or both of these systems.3 respiration?
415
Cardiac Causes of Dyspnea
What are the The mechanisms of dyspnea in patients with heart disease are diverse, and there is extensive overlap with the pulmonary mechanisms system. Aerobic respiration depends on the delivery of deoxygenated blood to the lungs and oxygenated blood to the of dyspnea peripheral tissues. These demands increase during exercise: When the heart is unable to adequately respond, metabolic related to derangements such as tissue acidosis develop, which stimulate the respiratory centers via local metaboreceptors. heart Additionally, pulmonary congestion related to elevated left atrial pressure directly stimulates local vascular and interstitial disease? receptors (eg, C-fibers, also called juxtapulmonary or J-receptors), which prompt the respiratory centers to increase respiratory drive (see Figure 20-2). Acute pulmonary edema is the most dramatic manifestation of pulmonary congestion, which impairs gas exchange and may trigger bronchoconstriction (known as cardiac asthma).1,2,5 What Oxygen delivery to the tissues is a function of the oxygen content of arterial blood (Cao2) and cardiac output (CO).6Do2 formula describes (mL/min) =Cao2 (mL/L) ×CO (L/min) oxygen delivery (Do2) to the tissues? What are the Cardiac output to the peripheral tissues is equal to the forward stroke volume (SV) of the left ventricle per beat multiplied main by heart rate (HR).3CO =SV ×HR determinants of cardiac output?
What are the cardiac causes of dyspnea?
Electrocardiograms and event Cardiac dysrhythmia. monitors are often helpful in diagnosing this group of conditions. A 48-year-old woman with Myocardial ischemia (angina equivalent). hypertension and diabetes mellitus presents with chronic exertional dyspnea; an exercise stress echocardiogram demonstrates inducible wall motion abnormalities. A disease of heart muscle that can be Cardiomyopathy. caused by a variety of conditions, such as myocardial ischemia, toxins (eg, alcohol), valvular disease, long-standing hypertension, myocarditis, and chronic tachyarrhythmia. A 72-year-old man presents with Aortic stenosis. exertional dyspnea and is found to have a late-peaking crescendo-decrescendo systolic murmur best heard over the right upper sternal border with radiation to the clavicle. Pulsus paradoxus in a patient with an Cardiac tamponade. enlarged cardiac silhouette on chest radiograph. Elevated JVP with Kussmaul’s sign (a Constrictive pericarditis. rise in JVP with inspiration) and Friedreich’s sign (a sharp and deep Y descent of the jugular venous waveform). Most often caused by viral infection, Myocarditis. this condition results in dilated cardiomyopathy in some patients. Impaired venous return to the heart Superior vena cava (SVC) syndrome. from the upper body. May be diagnosed using Intracardiac shunt. echocardiography with agitated saline contrast.
Which cardiac dysrhythmia might Second-degree Atrioventricular (AV) block is characterized electrocardiographically by the
416
present with exertional dyspnea and the electrocardiographic presence of dropped QRS complexes in an unpredictable pattern?
presence of both conducted beats (ie, P wave followed by an associated QRS complex) and nonconducted (or dropped) beats (ie, P wave not followed by an associated QRS complex). Mobitz II second-degree AV block, which is often symptomatic, is defined by the presence of nonconducted beats that do not occur in a predictable-pattern (see Figure 3-4). Exercise typically worsens conduction in the setting of Mobitz II second degree AV block. 7
What is an Chronic stable angina describes chest or arm discomfort that is reproducible with physical exertion or emotional stress, angina and is relieved promptly (in <5 minutes) with rest or the use of sublingual nitroglycerin. Some patients (particularly equivalent? women, the elderly, and diabetics) may experience other symptoms that are associated with exertion or stress, and are relieved with rest or sublingual nitroglycerin. These symptoms should be considered equivalent to angina. Exertional dyspnea is the most common angina equivalent symptom; others include isolated discomfort in the jaw, neck, ear, arm, shoulder, back, or epigastric region, nausea, vomiting, diaphoresis, and unexplained fatigue. Patients with acute coronary syndrome (eg, non-ST elevation myocardial infarction) may also present with angina equivalent symptoms.8 What are the Patients with heart failure frequently complain of dyspnea on exertion, orthopnea, and paroxysmal nocturnal dyspnea. characteristics Orthopnea describes dyspnea that occurs in the recumbent position. Paroxysmal nocturnal dyspnea describes dyspnea of dyspnea in that awakens the patient from sleep, typically after 1 or 2 hours, and often abates in the upright position.2 patients with cardiomyopathy who develop heart failure? What are the Aortic stenosis is typically asymptomatic until it becomes severe. Exertional dyspnea is one of the most common mechanisms of symptoms in patients with severe aortic stenosis. It occurs when concentric left ventricular hypertrophy leads to dyspnea in elevated end-diastolic pressure with associated pulmonary venous hypertension, reduced cardiac output from diastolic patients with dysfunction, or myocardial ischemia from Increased oxygen demand and reduced coronary flow reserve even in the aortic stenosis? absence of coronary artery disease (dyspnea in this context is an angina equivalent). Once overt systolic heart failure develops, mean survival is 2 years in patients who do not receive treatment.9,10 What is cardiac Cardiac tamponade describes a reduction in cardiac output as a consequence of compression from pericardial fluid, pus, tamponade? blood, or gas, either alone or in combination (see Figure 5-2). Virtually any cause of pericarditis can result in cardiac tamponade. Dyspnea on exertion is a common symptom in patients with tamponade physiology, typically progressing to air hunger at rest. Orthopnea is occasionally present. Intravenous fluids can be beneficial to the hypovolemic patient with cardiac tamponade; however, definitive treatment requires urgent evacuation of the pericardial space.11 What are the Constrictive pericarditis can develop following virtually any cause of acute pericarditis. The most common causes in the most common industrialized world include infectious pericarditis (most often viral), cardiac or pericardial surgery, and external beam causes of radiation to the mediastinum. Dyspnea becomes more prominent at higher central diastolic pressures (ie, >15 mm Hg). constrictive Orthopnea is occasionally present.12 pericarditis in the industrialized world? What symptoms Myocarditis is associated with a wide spectrum of clinical presentations, from asymptomatic disease to fulminant heart are associated failure. Dyspnea and chest pain are among the most common presenting complaints. Some patients experience with complete recovery, whereas others develop chronic dilated cardiomyopathy and progressive heart failure, necessitating myocarditis? transplantation.13 What is superior SVC syndrome occurs when obstruction of the SVC results in impaired cardiac filling, giving rise to a constellation of vena cava characteristic symptoms and signs. Dyspnea is one of the most frequent symptoms; orthopnea occurs in around one-half syndrome? of patients. Distended neck veins, distended superficial chest veins, and edema of the face, neck, and arms are among the most common signs.14 What is the Significant right-to-left intracardiac shunt causes hypoxemia that is sensed by central and peripheral chemoreceptors mechanism of and relayed to the respiratory centers in the brainstem, which respond by increasing respiratory drive (see Figure 20-2). dyspnea in Because hypoxemia caused by anatomic shunt cannot be corrected with Increased ventilation, afferent/efferent patients with mismatch sensed by the respiratory centers results in dyspnea. right-to-left intracardiac shunt?
417
Pulmonary Causes of Dyspnea
What are the mechanisms of dyspnea related to pulmonary disease? What are the determinants of effective gas exchange in the lungs? The pulmonary causes of dyspnea can be separated into which anatomic subcategories?
Aerobic respiration depends on the exchange of carbon dioxide and oxygen in the lungs. When this function is impaired, patients may develop hypercarbia or hypoxemia. These metabolic derangements are sensed by the central and peripheral chemoreceptors, which then provide stimulatory feedback to the respiratory centers. If the lungs are unable to satisfy the central demand for improved gas exchange, then the patient will experience dyspnea. In addition, some pulmonary diseases (eg, pulmonary edema) activate local receptors (eg, pulmonary C-fibers), which stimulate the respiratory centers and may contribute to the sensation of dyspnea (see Figure 20-2). Gas exchange in the lungs is determined by the balance between pulmonary ventilation (V) and capillary blood flow (Q). A perfect match between ventilation and perfusion (ie, V/Q of 1) is the reference point for defining normal and abnormal gas exchange in the lungs. Dead space occurs when there is excess alveolar ventilation relative to pulmonary capillary perfusion (ie, V/Q >1). Shunt occurs when there is excess pulmonary capillary perfusion relative to alveolar ventilation (ie, V/Q <1) (see Figure 46-6).6 The pulmonary causes of dyspnea can be separated into the following subcategories: airway, parenchyma, vasculature, and pleura.
418
Dyspnea Related to the Airway
What are the causes of dyspnea related to the airway?
Self-limited inflammation of the large airways. A 63-year-old man with an extensive smoking history requires hospitalization at least once per year for episodes of Increased cough with sputum production and dyspnea. A 26-year-old man experiences episodes of dyspnea, chest tightness, and wheezing triggered by cold air, dust, and perfume. Airway dilation and thickening on chest imaging (Figure 20-3).
Acute bronchitis. Chronic obstructive pulmonary disease.
Asthma.
Bronchiectasis.
FIGURE 20-3 Bronchiectasis in a patient with cystic fibrosis. A dilated bronchus in the right upper lobe (large
A 36-year-old man develops acute dyspnea, stridor, and hives during dinner at a seafood restaurant. Dynamic airway collapse on expiratory chest imaging. Acute upper airway obstruction that may be visible on chest imaging.
What causes acute bronchitis in adults?
What is the pharmacologic
arrows) shows a lack of tapering. The signet ring sign (small arrow) describes the cross-sectional appearance of a dilated bronchus compared with the pulmonary artery branch that accompanies it (these structures are normally the same size). (From Webb WR, Higgins CB. Thoracic Imaging Pulmonary and Cardiovascular Radiology. 3rd ed. Philadelphia, PA: Wolters Kluwer Health; 2017.
Anaphylaxis.
Tracheomalacia.
Foreign body aspiration.
Acute bronchitis is most often caused by viral infection. Cough is the predominant symptom and can be either productive or nonproductive. Dyspnea and wheezing may also be present. The condition is self-limited but often protracted, with a mean duration of about 24 days. Treatment is supportive in most cases. Antimicrobial agents may be useful in select circumstances, such as influenza and Bordetella pertussis infections. Airway irritants such as smoke can also cause acute bronchitis.15 Exacerbations of COPD are characterized by periods of Increased dyspnea and cough with sputum
419
treatment for acute exacerbations of chronic obstructive pulmonary disease?
What is the key spirometric finding in patients with asthma? What are the clinical manifestations of bronchiectasis?
What is the pharmacologic treatment of choice for anaphylaxis?
What are the most common causes of tracheomalacia in adults?
Which lung is most often affected by the aspiration of foreign material?
production. Common triggers include viral or bacterial lower respiratory tract infection, heart failure, pulmonary embolism (PE), nonadherence to inhaler therapy, and inhalation of irritants such as tobacco smoke or particles. Exacerbations that are severe enough to require hospitalization are typically treated with inhaled short-acting bronchodilators (eg, albuterol) and systemic glucocorticoids (eg, prednisone) with or without antibiotics (eg, doxycycline). 16 Asthma is associated with reversible airway obstruction on spirometric testing. Reversibility is the key feature that distinguishes asthma from COPD and other causes of obstructive lung disease. The vast majority of patients with bronchiectasis have chronic cough productive of sputum that is thick and mucoid. Dyspnea occurs in most patients. Common physical findings include rales, wheezing, and rhonchi. Patients with acute exacerbations of bronchiectasis experience fever and Increased dyspnea, wheezing, cough, and sputum production. Antibiotics are the mainstay of treatment. Sputum culture can be helpful in guiding therapy.17 Anaphylaxis is a life-threatening systemic hypersensitivity reaction that typically develops within 5 to 30 minutes of antigen exposure. Pulmonary manifestations occur as a result of bronchospasm and include dyspnea, wheezing, cough, and chest tightness. Securing the airway is critical, sometimes requiring intubation. Intramuscular epinephrine should be administered to all patients suspected of having anaphylaxis. It relieves respiratory distress by inducing bronchodilation, with an onset of action of 3 to 5 minutes.18 Tracheomalacia refers to weakness of the tracheal wall such that it is softer and susceptible to collapse, particularly on expiration. Tracheobronchomalacia refers to the combination of tracheal and mainstem bronchial involvement, whereas bronchomalacia refers to isolated involvement of the mainstem bronchi. The most common acquired causes in adults include tracheostomy and intubation with an endobronchial tube. Chronic inflammation from inhaled irritants such as tobacco smoke is an important risk factor. Symptoms most often include dyspnea, cough (described as “barking” in quality), sputum production, and hemoptysis.19 The right mainstem bronchus is wider and more vertically oriented than the left, making the right lung more susceptible to foreign body aspiration. Men aspirate foreign bodies more frequently than women. Common symptoms include dyspnea, wheezing, and cough; some patients may present with acute respiratory failure. Flexible bronchoscopy is both diagnostic and therapeutic.20
420
Dyspnea Related to the Pulmonary Parenchyma
What are the causes of dyspnea related to the pulmonary parenchyma?
Fever and purulent cough, with focal rales and egophony on auscultation of the lung. Chest radiography typically demonstrates bilateral ground glass opacities, vascular indistinctness, and septal thickening in the periphery. This condition can be associated with Decreased chest excursion on the affected side with inspiratory rales that clear after deep breathing or cough. A 72-year-old woman with an extensive smoking history presents with chronic dyspnea and is found to have flattened diaphragms and hyperlucency of the lower lungs on chest radiograph (Figure 20-4).
Pneumonia.
Pulmonary edema.
Atelectasis.21
Emphysema (a subtype of COPD).
FIGURE 20-4 Posteroanterior (A) and lateral (B) chest radiographs of a patient with emphysema show
A 65-year-old man with COPD and active cigarette use presents with weight loss, dyspnea, and hemoptysis. Digital clubbing and bilateral fine end-inspiratory rales on auscultation of the chest.
How can tactile fremitus aid in the diagnosis of pneumonia?
What are the mechanisms of dyspnea in patients with pulmonary edema?
Which group of hospitalized patients is at high risk for atelectasis? What are the key pulmonary function test findings in
flattening of the diaphragm with blunting of the costophrenic angles, hyperlucency in the lower lungs, Increased retrosternal lucency, Increased anteroposterior diameter of the chest, and enlarged central pulmonary arteries. (From Collins J, Stern EJ. Chest Radiology: The Essentials. Philadelphia, PA: Wolters Kluwer Health. 2015.
Lung cancer.
Interstitial lung disease (ILD) (eg, idiopathic pulmonary fibrosis).
Dullness to percussion is typical over an area of consolidated lung but also occurs over a pleural effusion. Tactile fremitus can be helpful in distinguishing between the 2 conditions. Tactile fremitus is Increased over an area of consolidation, whereas it is Decreased (often absent) over an effusion. The area of compressed lung just above an effusion is sometimes associated with a thin band of Increased tactile fremitus and other signs of consolidation.22 Pulmonary edema may be caused by left heart failure (ie, cardiogenic), or injury to the endothelial and epithelial barriers (ie, noncardiogenic). It leads to impaired gas exchange in the form of physiological shunt and activation of local interstitial receptors (C-fibers) that stimulate the respiratory centers (see Figure 20- 2).2,23 Atelectasis is common in postoperative patients and can lead to dyspnea, cough, tachypnea, hypoxemia, and even acute respiratory failure. Early ambulation, breathing exercises, and incentive spirometry may be helpful in the prevention and treatment of atelectasis in this population.21 COPD describes chronic airflow limitation, typically as a result of small airway disease and parenchymal destruction. Airway disease predominates in some patients producing the chronic bronchitis phenotype
421
How common is dyspnea in patients with lung cancer?
What are the key pulmonary function test findings in patients with interstitial lung disease?
phenotype (the “pink puffer”). Pulmonary function studies differ between the two. Both are associateds additionally associated with Increased total lung capacity (mostly as a result of Increased residual volume) 1 1 with Decreased FEV and Decreased ratio between FEV and forced vital capacity (FVC). Emphysema i and Decreased diffusing capacity for carbon monoxide (D ). lco 24 Dyspnea is common in all stages of lung cancer. It is distressing to patients and interferes with activities of daily living. Mechanisms are diverse, including direct tumor invasion, associated pleural effusion, PE, anemia, and treatment-related complications (eg, radiation pneumonitis).25 ILD is a restrictive lung disease. Spirometry demonstrates Decreased FEV1 with a preserved or Increased FEV1/FVC ratio; total lung capacity is Decreased; and Dlco is Decreased.
422
Dyspnea Related to the Pulmonary Vasculature
What are the causes of dyspnea related to the pulmonary vasculature?
A 68-year-old woman with pancreatic cancer develops new dyspnea and hemoptysis; electrocardiogram demonstrates sinus tachycardia, T-wave inversions in leads V1- V3, and right axis deviation. Loud and palpable P2 in a patient with ILD. A 44-year-old woman with a history of chronic sinusitis presents with dyspnea and hemoptysis and is found to have hematuria and Acute Kidney Injury with dysmorphic red blood cells (see Figure 34-4) and red blood cell casts on urine sediment analysis. A patient with hereditary hemorrhagic telangiectasia develops dyspnea and hypoxemia that does not correct with 100% inhaled oxygen.
Pulmonary embolism.
Pulmonary hypertension. Granulomatosis with polyangiitis (GPA, or Wegener’s granulomatosis).
Pulmonary arteriovenous malformation.
What are the characteristics of dyspnea Most patients with PE experience acute dyspnea on exertion or at rest. Onset is typically related to pulmonary embolism? rapid, within seconds to hours. Orthopnea is frequently present. The vast majority of patients have either dyspnea or tachypnea on presentation. Other common symptoms include pleuritic chest pain, cough, and hemoptysis.26 What are the mechanisms of dyspnea in Dyspnea on exertion is a common symptom of pulmonary hypertension. Mechanisms pulmonary hypertension? include impaired gas exchange related to Increased dead space and Decreased Dlco, stimulation of local vascular receptors (eg, C-fibers), opening of a right-to-left intracardiac shunt, and right heart failure (ie, cor pulmonale).27 Which systemic vasculitides are most often Diffuse alveolar hemorrhage (DAH) is a life-threatening condition that typically presents associated with pulmonary-renal syndrome? with dyspnea, cough, and hemoptysis. Pulmonary-renal syndrome refers to the combination of DAH and glomerulonephritis. It is most frequently caused by the antineutrophil cytoplasmic antibody (ANCA)–associated small vessel vasculitides (eg, microscopic polyangiitis), antiglomerular basement membrane disease, or vasculitis associated with systemic disease, particularly systemic lupus erythematosus.28 What condition should be suspected in a Hepatopulmonary syndrome describes the development of pulmonary arteriovenous cirrhotic patient who develops dyspnea that malformations at the lung bases in patients with advanced liver disease. The upright worsens in the upright position and position augments blood flow to the lung bases, which increases shunt fraction, whereas improves while lying flat? the supine position decreases shunt fraction. This physiology causes dyspnea in the upright position (platypnea), as well as arterial desaturation (orthodeoxia), both of which improve with recumbency.
423
Dyspnea Related to the Pleura
What are the causes of dyspnea related to the pleura?
Dullness to percussion of the chest with Decreased tactile fremitus. Hyperresonance to percussion of one side of the chest.
What is the treatment for the dyspnea associated with pleural effusion? Which patients are at highest risk for spontaneous pneumothorax?
Pleural effusion.
Pneumothorax.
Treatment strategies for pleural effusions depend on the size, nature of the fluid, and symptom burden. For example, small to moderate size transudative pleural effusions caused by heart failure are often responsive to diuretic therapy. Large and symptomatic malignant pleural effusions, on the other hand, are generally best managed with drainage (ie, thoracentesis) along with treatment for the underlying malignancy (if possible). Patients often experience immediate relief when large pleural effusions are drained. Pneumothorax describes the presence of air in the pleural space (see Figure 24-3). Symptoms include dyspnea and chest pain, the severity of which is generally proportional to the size of the pneumothorax. Primary spontaneous pneumothorax occurs in patients without underlying lung disease. The prototypical patient is a tall, thin male between 10 and 30 years of age who actively smokes or has a history of smoking. Secondary spontaneous pneumothorax occurs in patients with underlying lung disease (eg, COPD, cystic fibrosis).29
424
Dyspnea Related to Other Causes
What are the other causes of dyspnea?
A young woman with agoraphobia suffers from recurrent episodes of dyspnea and chest pain. Conjunctival pallor. A traveling soccer team unexpectedly loses to a weaker home team in La Paz, Bolivia, a city that is 3600 m above sea level. A 38-year-old woman presents with double vision and drooping eyelids that are more noticeable at the end of the day, and is found to have a mediastinal mass. Respiratory compensation. Women are at risk. An Endocrinopathy. Associated with a sedentary lifestyle or prolonged hospitalization.
Is there an association between cardiopulmonary disease and anxiety?
What is the mechanism of dyspnea in patients with anemia?
What life-threatening condition should be considered in a climber who develops dyspnea and pink frothy sputum at the summit of a mountain?
What extrapulmonary causes of hypoventilation are associated with dyspnea? In a patient with pure metabolic acidosis and serum bicarbonate (HCO3¯) concentration of 10 mEq/L, what is the expected partial pressure of carbon dioxide (Paco2) after respiratory compensation? When does dyspnea occur during the course of a normal pregnancy? What is the mechanism of dyspnea in patients with thyrotoxicosis?
Panic attack.
Anemia. Reduced partial pressure of inspired oxygen (PIo2) caused by high altitude.
Myasthenia gravis (associated with thymoma).
Metabolic acidosis. Pregnancy. Thyrotoxicosis. Deconditioning.
Patients with underlying cardiopulmonary disease, particularly obstructive lung disease, experience a high rate of anxiety, which can exacerbate existing chronic dyspnea. It is important to identify anxiety and panic disorders in this population, as treatment can improve functional status and quality of life. Options for patients with respiratory disease include nonpharmacologic and limited pharmacologic (eg, selective serotonin reuptake inhibitors, buspirone) treatment. Benzodiazepines and other respiratory depressants should generally be avoided unless patients are receiving hospice care.30 Hemoglobin concentration is one of the key determinants of the oxygen content in arterial blood (see chapter 46, Hypoxemia). Anemia causes impaired oxygen delivery to the tissues, resulting in metabolic derangements such as tissue acidosis, which stimulate the respiratory centers via local metaboreceptors (see Figure 20-2).1 High-altitude pulmonary edema (HAPE) is a life-threatening condition that most commonly develops within 2 to 4 days of ascending 2500 m or more above sea level. It is responsible for the majority of deaths related to high altitude. Slow ascent to allow for acclimatization is the most effective way to prevent HAPE; nifedipine is the pharmacologic agent of choice for prevention. If it develops, patients should be treated with supplemental oxygen, advised to rest, and immediately evacuated to lower altitudes in severe cases.31 Extrapulmonary causes of hypoventilation associated with dyspnea include obesity, ascites, neuromuscular diseases (eg, amyotrophic lateral sclerosis, myasthenia gravis, Guillain-Barré syndrome), and chest wall deformities (eg, pectus excavatum). Metabolic acidosis generates a compensatory increase in ventilation with a target Paco2 lower than the normal 40 mm Hg in arterial blood. Winters’ formula can be used to calculate the expected Paco2: predicted Paco2 =(1.5 ×[HCO3¯]) +8 ± 2.In a patient with normal lung function and a serum HCO3¯ of 10 mEq/L, predicted Paco2 =(1.5 ×10) +8 ± 2 =23 ± 2 mm Hg.32
Dyspnea is a common feature of normal pregnancy. It can begin in the first trimester and occurs in most patients by the end of the second trimester. A variety of mechanisms are involved, including an increase in respiratory drive mediated by estrogen and progesterone.33 Patients with thyrotoxicosis frequently experience dyspnea, particularly with exertion. It occurs as a result of exaggerated central respiratory drive in response to hypercapnia and hypoxemia. Severity correlates with the degree of thyrotoxicosis. β-Blocker therapy is often effective at mitigating dyspnea until euthyroidism can be achieved. Other mechanisms of dyspnea in patients with thyrotoxicosis include respiratory muscle weakness and pulmonary hypertension, both of which are reversible with management of thyrotoxicosis. 34,35
425
dyspnea in patients with respiratory centers via local metaboreceptors (see Figure 20-2). Patients with cardiopulmonary disease deconditioning? and chronic dyspnea are at risk for becoming sedentary and developing deconditioning, which can worsen symptoms. Pulmonary rehabilitation and exercise training can be helpful. 1
426 Case Summary A 71-year-old woman with an extensive smoking history presents elevated JVP, dilated superficial veins on the chest, an abnormal with progressive dyspnea and weight loss, and is found to have cranial nerve examination, and an apical mass on chest imaging.
What is the most likely diagnosis in this patient? Superior vena cava syndrome.
427
Bonus Questions
case are suggestive of syndrome in this case include elevated JVP, dilated superficial veins ofithe chest (see Figure 20-1B), edema of the face, neck, and upper extremities, and the C What features in this The patient in this case has classic symptoms of SVC syndrome, includ ng facial and neck swelling, dyspnea, and cough. Physical findings suggestive of SV superior vena cava onset of facial plethora after raising the arms above the head (Pemberton’s sign). syndrome? What are the causes of Most cases of SVC syndrome in the industrialized world are related to intrathoracic malignancy (eg, lung cancer, mediastinal lymphoma); other causes include superior vena cava SVC stenosis or thrombosis (usually caused by intravascular devices), goiter, aortic aneurysm, and fibrosing mediastinitis. 14 syndrome? Which cranial nerve Ptosis and miosis of the left eye are demonstrated in Figure 20-1A. abnormalities are evident in Figure 20-1A? of the combination ofhis sympathetic innervationiof the eye (anhidrosis rounds out the classic triad). The sympathetic neurons originate in the hypothalamus, descend into the cervical What is the significance The combination of ptos s and miosis in this case is suggestive of Horner’s syndrome, which describes a constellation of findings caused by disruption of the ptosis and miosis in t spinal cord, travel across the pulmonary apex, and travel up the wall of the internal carotid artery before joining the ophthalmic division of the trigeminal case? nerve. Disruption of the sympathetic neurons at any point along this pathway will result in ipsilateral Horner’s syndrome. 36 What is the most likely The smoking history, weight loss, and apical mass on chest imaging in this case suggest an apical lung tumor (Pancoast tumor), which is likely causing cava obstruction and cause of superior vena external compression of the SVC as well as disruption of the sympathetic neurons that travel across the pulmonary apex. Horner’s syndrome in this case? What is the treatment Symptoms of SVC syndrome related to extrinsic compression from lung cancer generally improve with chemotherapy and local radiation. If there is for superior vena cavang associated SVC thrombosis, anticoagulation should be started. Endovascular revascularization techniques (eg, stenting) or open surgical procedures may also cancer? syndrome related to lu be considered. 14
428 Key Points
Dyspnea is the subjective experience of breathing discomfort or Breathing is regulated by the respiratory centers in the brainstem, chemoreceptors, mechanoreceptors, andimetaboreceptors. anatomic subcategories: airway, parenchyma, vasculature,landng breathlessness. which receive information from various sensory inputs, including The causes of dyspnea can be separated nto the following categories: cardiac, pulmonary, and other. Most causes of dyspnea are related to the heart and/or the ungs. Pulmonary causes of dyspnea can be separated into the followi pleura.
429
References 1. Parshall MB, Schwartzstein RM, Adams L, et al. An official American Thoracic Society statement: update on the mechanisms, assessment, and management of dyspnea. Am J
Respir Crit Care Med. 2012;185(4):435-452. 2. WalkertHK, Hall WD, Hurst JW, eds. Clinical Methods: The History, Physical, and Labora ory Examinations. 3rd ed. Boston: Butterworths; 1990. 3. Berne RML, Levy MN. Physiology. 4th ed. St. Louis, MO: Mosby, Inc.; 1998. 4. Burki NK, Lee LY. Mechanisms of dyspnea. Chest. 2010;138(5):1196-1201. 5. Witte KK, Clark AL. Why does chronic heart failure cause breathlessness and fatigue? Prog Cardiovasc Dis. 2007;49(5):366-384.
-
Marino PL. The ICU Book. 3rd ed. Philadelphia, PA: Lippincott Williams & Wilkins—a Wolters Kluwer business; 2007.
-
Merideth J, Pruitt RD. Cardiac arrhythmias. 5. Disturbances in cardiac conduction and their management. Circulation. 1973;47(5):1098-1107. Anderson JL, Adams CD, Antman EM, et al. 2011 ACCF/AHA focused update
-
incorporated into the ACC/AHA 2007 guidelines for the management of patients with College of Cardiology Foundation/American Heart Association Task Force on practice unstable angina/non-ST-elevation myocardial infarction: a report of the American guidelines. Circulation. 2011;123(18):e426-e579.
-
Maganti K, Rigolin VH, Sarano ME, Bonow RO. Valvular heart disease: diagnosis and management. Mayo Clin Proc. 2010;85(5):483-500.
-
Ross Jr J, Braunwald E. Aortic stenosis. Circulation. 1968;38(1 suppl):61-67.
-
Spodick DH. The.Pericardium: A Comprehensive Textbook. New York, NY: Marcel 12. Schwefer M, Aschenbach R, Heidemann J, Mey C, Lapp H.lConstrictive pericarditis, Dekker, Inc.; 1997 still a diagnostic challenge: comprehensive review of clinica management. Eur J Cardio Thorac Surg. 2009;36(3):502-510.
-
Schultz JC,.Hilliard AA, Cooper Jr LT, Rihal CS. Diagnosis and treatment of viral myocarditis Mayo Clin Proc. 2009;84(11):1001-1009.
-
Cheng S. Superior vena cava syndrome: a contemporary review of a historic disease. Cardiol Rev. 2009;17(1):16-23.
-
Wenzel(RP, Fowler AA, 3rd. Clinical practice. Acute bronchitis. N Engl J Med. 16. Decramer M, Janssens W, Miravitlles M. Chronic obstructive pulmonary disease. 2006;355 20):2125-2130. Lancet. 2012;379(9823):1341-1351.
-
Barker AF. Bronchiectasis. N Engl J Med. 2002;346(18):1383-1393.
- Arnold JJ, Williams PM. Anaphylaxis: recognition and management. Am Fam Physician. 2011;84(10):1111-1118. . Carden KA, Boiselle PM, Waltz DA, Ernst A. Tracheomalacia and
19tracheobronchomalacia in children and adults: an in-depth review. Chest. 2005;127(3):984-1005.
- Sehgal IS, Dhooria S, Ram B, et al. Foreign body inhalation in the adult population: experience of 25,998 bronchoscopies and systematic review of the literature. Respir
Care. 2015;60(10):1438-1448. 21treatment of perioperative pulmonary atelectasis. Expert ReviRespir Med.t2015;9(1):97- . Restrepo RD, Braverman J. Current challenges in the recogn tion, preven ion and
430
107. 22. Sapira JD. The Art & Science of Bedside Diagnosis. Baltimore, MD: Urban & Schwarzenberg, Inc.; 1990.
-
Murray JF. Pulmonary edema: pathophysiology and diagnosis. Int J Tuberc Lung Dis. 2011;15(2):155-160, i.
-
Ranu H, Wilde M, Madden B. Pulmonary function tests. Ulster Med J. 2011;80(2):84- 90.
-
Williams AC, GrantiM, Tiep B, Kim JY, HayterlJ. Dyspnea management in early stage 26. Stein PD, Beemath A, Matta F, et al. Clinical characteristics of patients with acute lung cancer: a palliat ve perspective. J Hosp Pal iat Nurs. 2012;14(5):341-342. pulmonary embolism: data from PIOPED II. Am J Med. 2007;120(10):871-879.
-
Sajkov D, Petrovsky N, Palange P. Management of dyspnea in advanced pulmonary arterial hypertension. Curr Opin Support Palliat Care. 2010;4(2):76-84. . McCabe C, Jones Q, Nikolopoulou A, Wathen C, Luqmani R. Pulmonary-renal
28syndromes: an update for respiratory physicians. Respir Med. 2011;105(10):1413-1421. 29. Choi WI. Pneumothorax. Tuberc Respir Dis. 2014;76(3):99-104. 30dyspnea, and respiratory disease. Theoretical and clinical considerations. Am J Respir . Smoller JW, Pollack MH, Otto MW, Rosenbaum JF, Kradin RL. Panic anxiety, Crit Care Med. 1996;154(1):6-17.
- Bhagi.S, Srivastava S, Singh SB. High-altitude pulmonary edema: review. J Occup Health 2014;56(4):235-243.
- Albert MS, Dell RB, WinterstRW. Quantitative displacement of acid-base equilibrium in metabolic acidosis. Ann In ern Med. 1967;66(2):312-322.
- Lee SY, Chien DK, Huang CH, Shih SC, Lee WC, Chang WH. Dyspnea in pregnancy. Taiwan J Obstet Gynecol. 2017;56(4):432-436. . Small D, Gibbons W, Levy RD, de Lucas P, Gregory W, Cosio MG. Exertio
34dyspnea and ventilation in hyperthyroidism. Chest. 1992;101(5):1268-1273. nal 35. Thurnheer R, Jenni R, Russi EW, Greminger P, Speich R. Hyperthyroidism and pulmonary hypertension. J Intern Med. 1997;242(2):185-188.
- Kanagalingam S, Miller NR. Horner syndrome: clinical perspectives. Eye Brain. 2015;7:35-46.
431
CHAPTER 21
432
Fever of Unknown Origin
433 Case: A 37-year-old man with oral ulcers A previously healthy 37-year-old Syrian man is evaluated in the experienced a self-limited illness characterized by fever, abdominal 2 weeks later developed intermittent fever that coincided with right clinic for fever and joint pain. Approximately 8 weeks ago, he pain, and diarrhea. He recovered to his usual state of good health but knee discomfort. He was evaluated in the clinic and found to have a temperature of 38.4°C with warmth and swelling of the right knee.
Urinalysis, blood cultures, and chest radiograph were unrevealing.nt The symptoms in the right knee eventually resolved, but intermitte fever persisted. Another visit to the clinic was nondiagnostic. A few days later, he developed pain and swelling of the left knee. He also
complains of occasional burning when he urinates, and bilateral eye first-degree relative with ulcerative colitis and ankylosing spondylitis. 1B). The insertion sitestoflthe Achilles tendons are erythematous and -wSynovial fluid analysis from left knee arthrocentesis is notable for a (PCR) for Neisseria gonorrhoeae is negative. Tests for serum antinuclear antibody are negative. Urine PCR studies for Chlamydia trachomatis discomfort without vision changes. Family history is notable for a Temperature is 38.6°C. There is mild bilateral conjunctival erythema. There are pain ess ulcerative lesions of the oral mucosa (Figure 21-1A). The lef knee is warm with a large effusion (Figure 21 tender to palpation. There are no genital ulcers. hite blood cell count of 6134/µL (67% neutrophils) with a negative Gram stain and culture. Synovial fluid polymerase chain reaction antibody, rheumatoid factor, and anti–cyclic citrullinated peptide and Neisseria gonorrhoeae are negative.
FIGURE 21-1
What is the most likely diagnosis in this patient?
What is fever of unknown origin (FUO)?
How common is fever of unknown origin?
FUO is classically defined as an illness lasting longer than 3 weeks, with measured temperature >38.3°C on several occasions, and failure to reach a diagnosis despite 1 week of inpatient investigation. Modifications have been made to this definition to incorporate more modern styles of practice, adjusting the minimum duration of investigation to 3 days in the hospital or 3 outpatient visits.1-3 Fever is a ubiquitous problem that occurs in virtually all patients at some point during life. True FUO is uncommon; for example in 1 community hospital setting in the United States, it was present in approximately 1 out of every 75 cases that involved infectious disease specialists. Although FUO occurs infrequently, it is associated with a heavy
434
The causes of fever of unknown origin can be separated into which general categories?
What is the relative prevalence of each category of fever of unknown origin? What is the prognosis of fever of unknown origin?
burden: most hospitalized patients remain in-house for prolonged periods of time (often weeks to months) and undergo considerable diagnostic testing.4 The causes of FUO can be separated into the following categories: infectious, noninfectious inflammatory, malignant, and other.
The causes of FUO have evolved over time, and the prevalence of each category depends on host and geographic factors. In the modern industrialized world, infection accounts for approximately 25% of cases; noninfectious inflammatory conditions account for 25%; malignancy, 15%; and the remainder consist of miscellaneous or idiopathic causes (most cases in this group remain undiagnosed). FUO is more likely to be caused by an atypical presentation of a common disease than a typical presentation of a rare disease.1,5,6 The prognosis of FUO depends on the underlying cause. The prognosis of idiopathic FUO is excellent; most cases resolve spontaneously within a few weeks. Infectious causes of FUO are also generally associated with a favorable prognosis; most patients survive without significant morbidity. Patients with noninfectious inflammatory causes of FUO are most likely to experience persistent morbidity. The highest mortality rates are associated with FUO due to malignancy; the majority of patients are dead within 5 years.4,6
435
Infectious Causes of Fever of Unknown Origin
What historical information is important to obtain when considering infectious causes of fever of unknown origin?
Types of historical information pertinent to infectious causes of FUO include demographic (eg, country of origin), medical (eg, history of diverticulitis, recent dental procedures or antibiotic use, blood transfusion, presence of indwelling foreign material), and social (eg, sexual and drug behaviors; travel history; sick contacts; and zoonotic, recreational, dietary, and occupational exposures).2
What are the infectious causes of fever of unknown origin?
A 73-year-old man with a history of Intra-abdominal abscess. diverticulitis, including a recent bout 6 weeks ago that was treated with oral antibiotics, presents with several weeks of fever and is found to have focal tenderness to palpation over the left lower quadrant of the abdomen. A 47-year-old woman who emigrated Miliary tuberculosis (TB). from Iran to the United States presents with FUO and is found to have cholestatic liver injury, normal chest radiograph, and computed tomography (CT) imaging of the abdomen demonstrating hepatomegaly and multiple low-density micronodules throughout the liver. A 62-year-old man, who recently Culture-negative endocarditis. completed an empiric course of antibiotics for fever 6 weeks after a dental cleaning, now presents with persistent fever, weight loss, and night sweats and is found to have negative blood cultures. Careful physical examination reveals a decrescendo diastolic murmur heard best over the third intercostal space of the left sternal border. This infection is typically acquired via Osteomyelitis. contiguous spread or hematogenous seeding. Antibiotics are not helpful for this type of Viral infection. infection. A 42-year-old man presents with fever, Leptospirosis. headache, myalgias, conjunctival suffusion (Figure 21-2), Acute Kidney Injury, moderate hepatocellular liver injury, hyponatremia, and thrombocytopenia a few weeks after windsurfing on the Columbia River, a fresh water source.
FIGURE 21-2 Conjunctival suffusion with subconjunctival hemorrhage is suggestive of leptospirosis. (Reprinted with permission from Lin CY, Chiu NC, Lee CM. Leptospirosis after typhoon. Am J Trop Med Hyg. 2012;86:187. Copyright © 2012 by The American Society of Tropical Medicine and Hygiene.)
What are the characteristics of fever of unknown origin caused by intra-
Intra-abdominal abscess is the most common infectious cause of FUO in the industrialized world. Commonly affected sites include the liver, spleen, and intraperitoneal cavity. Most
436
abdominal abscess?
Under what circumstances is infection with Mycobacterium tuberculosis most likely to go undetected?
Under what circumstances is endocarditis most likely to present with negative blood cultures?
What is the most common site of osteomyelitis in patients presenting with fever of unknown origin?
What are the most common viral causes of fever of unknown origin?
Which zoonotic infection should be suspected in a previously healthy 18-year-old patient with a pet cat who presents with fever of unknown origin and tender regional lymphadenopathy?
patients have a compatible history, such as biliary disease, diverticulitis, appendicitis, or Crohn’s disease. Tenderness on examination is typically present; however, elderly patients tend to have more subtle symptoms and signs, which can result in a protracted course. CT imaging of the abdomen has a high diagnostic yield and should be one of the first studies performed in patients with FUO, particularly with a suggestive history or examination. Dental, renal, and perinephric abscess are other common sources of occult infection in patients with FUO. 5-7 Forms of TB most likely to present with FUO include extrapulmonary disease (eg, hepatic TB) without clear localizing features, miliary disease without the characteristic pattern on chest radiography, and pulmonary disease in patients with compromised immune systems (chest radiography may be normal in patients with acquired immunodeficiency syndrome). Key investigations include imaging to evaluate for subtle pulmonary or extrapulmonary disease, tuberculin skin testing or blood interferon-γ release assay, studies to identify Mycobacterium tuberculosis in samples from sputum or bronchoscopy with bronchoalveolar lavage (eg, smear, culture, PCR), and histopathologic evaluation of involved tissue (eg, lung, liver, bone marrow).7,8 Culture-negative endocarditis can occur with typical pathogens (eg, Streptococcus species) when antibiotics have been administered before culture is obtained. It can also occur with atypical organisms that do not grow in routine culture media, such as Bartonella species, Brucella species, Coxiella burnetii, Abiotrophia defectiva, Granulicatella species, Tropheryma whipplei, and fungi. The HACEK organisms (Haemophilus species, Aggregatibacter species, Cardiobacterium hominis, Eikenella corrodens, and Kingella species) will grow in culture if given sufficient time. Key investigations to evaluate for culture-negative endocarditis include careful physical examination (eg, cardiovascular, fundoscopic), special culturing techniques, serologic tests (eg, Coxiella), molecular techniques (eg, PCR), and echocardiography.7,9 The spine is the most common site of osteomyelitis in patients presenting with FUO. Local discomfort may be minimal. Magnetic resonance imaging (MRI) of the spine is helpful for identification. Infected joint prostheses are also a frequent source of osteomyelitis in patients with FUO, highlighting the importance of identifying the presence of foreign material in the body.5,10 Viral infections most frequently implicated in cases of FUO include cytomegalovirus (CMV), Epstein-Barr virus (EBV), and human immunodeficiency virus (HIV). These viruses can produce a protracted mononucleosis-like illness. One-quarter of immunocompetent patients with CMV infection have fever longer than 3 weeks. Key investigations include peripheral white blood cell count with differential, peripheral blood smear, liver biochemical tests, serologic tests, and molecular techniques.5,10 Infection with Bartonella henselae is an important cause of FUO in immunocompetent and immunocompromised patients. It is transmitted via a scratch or bite from an infected cat, or a bite from an infected cat flea. Classic cat scratch disease (fever and regional lymphadenopathy lasting <7 days) is the most common clinical manifestation. However, when presenting as FUO, cat scratch disease can occur with or without regional lymphadenopathy. The diagnosis is usually made with serologic or molecular techniques. A number of zoonotic infections can present with FUO, including borreliosis, bartonellosis, Rocky Mountain spotted fever, Q fever, ehrlichiosis, tularemia, leptospirosis, and brucellosis.11,12
437
Noninfectious Inflammatory Causes of Fever of Unknown Origin
What historical information is important to obtain when considering noninfectious inflammatory causes of fever of unknown origin?
Historical information pertinent to noninfectious inflammatory causes of FUO include a history of recent illness (eg, gastroenteritis), morning stiffness lasting >1 hour, prominent arthralgias or myalgias, ocular symptoms, recurrent or persistent skin rash, change in bowel patterns, response to glucocorticoid therapy during the course of illness, and familial disorders. Because noninfectious inflammatory conditions often result in systemic manifestations, a thorough review of systems is important.7
What are the noninfectious inflammatory causes of fever of unknown origin?
Dramatic elevation in serum ferritin, often
2000 ng/mL. Classically presents with symmetric polyarticular inflammatory arthritis, most often involving the wrist, metacarpophalangeal, and proximal interphalangeal joints. A 32-year-old woman with several weeks of intermittent fever, pancytopenia, and low serum complement levels. A 29-year-old man with multiple recent sexual partners presents with fever and sterile inflammatory arthritis of the right knee several weeks after being treated for dysuria with urethral discharge. Bilateral hilar lymphadenopathy on chest imaging. A 47-year-old man with chronic hepatitis B infection, testicular pain, and livedo reticularis. Associated with giant cell arteritis (GCA). Colonoscopy is necessary for diagnosis. A 34-year-old Lebanese man has experienced recurrent episodes of fever, abdominal pain, and inflammatory arthritis since he was 8 years of age.
What is adult-onset Still’s disease?
Adult-onset Still’s disease (AOSD).13
Rheumatoid arthritis (RA).
Systemic lupus erythematosus (SLE).
Reactive arthritis.
Sarcoidosis.
Polyarteritis nodosa.
Polymyalgia rheumatica (PMR). Inflammatory bowel disease (IBD). Familial Mediterranean fever (FMF).
AOSD is a systemic inflammatory disorder of unknown etiology. It occurs worldwide and is associated with a bimodal age distribution with peaks between the ages 15 to 25 years and 36 to 46 years. Typical clinical manifestations include fever, nonsuppurative pharyngitis, arthralgias/arthritis, and a transient, evanescent, salmon-colored macular or maculopapular skin rash. The fever is usually high (temperature ≥39°C) and quotidian (recurring daily) or double quotidian (2 peaks daily). The rash often appears when the patient is
438
What diagnostic studies are helpful in evaluating for rheumatoid arthritis in patients with fever of unknown origin?
Which laboratory studies are helpful in evaluating for systemic lupus erythematosus in patients with fever of unknown origin? Which general types of infection are most often associated with reactive arthritis? What diagnostic studies are helpful in evaluating for sarcoidosis in patients with fever of unknown origin?
febrile and disappears during afebrile periods. AOSD presents as FUO in up to 10% of patients. Markedly elevated serum ferritin levels (often >2000 ng/mL) are characteristic. Glucocorticoids are generally first-line treatment. 13,14 In patients with FUO, the presence of serum rheumatoid factor or anti–cyclic citrullinated peptide antibodies can be suggestive of RA. Radiographs of the hands and feet may reveal joint space narrowing and erosions of the bone (Figure 21-3).14
FIGURE 21-3 Bone erosions (arrows) at the metacarpophalangeal joints in a patient with rheumatoid arthritis. (From Greenspan A. Orthopedic Imaging: A Practical Approach. 5th ed. Philadelphia, PA: Lippincott Williams & Wilkins; 2011.)
In patients with FUO, the presence of leukopenia, serum antinuclear antibodies (ANA), double-stranded DNA antibodies, or low serum complement levels can be suggestive of SLE. There is a significant false positive ANA rate in the general FUO population.11,14
Infections of the genitourinary and Gastrointestinal tracts are most frequently associated with the development of reactive arthritis.15
The presence of hilar lymphadenopathy on chest imaging (Figure 21-4) or granulomas on histopathologic evaluation of involved tissue (eg, lymph node, liver) can be suggestive of sarcoidosis. In general, excisional lymph node biopsy is associated with higher yield compared with needle aspiration. The posterior cervical, epitrochlear, supraclavicular, hilar, mediastinal, and retroperitoneal lymph nodes are most likely to provide a diagnosis in patients with FUO. The anterior cervical, axillary, and inguinal nodes are more likely to yield nonspecific information.2,14
FIGURE 21-4 Posteroanterior (PA) chest radiograph showing right paratracheal (arrowheads) and bilateral hilar
What type of systemic vasculitis is a frequent cause of fever of unknown origin in elderly patients? What test can confirm the diagnosis of polymyalgia rheumatica?
Which type of inflammatory bowel disease is most likely to present with fever of uknown origin?
What is familial Mediterranean fever?
(arrows) lymphadenopathy, classic for sarcoidosis. (From Collins J, Stern EJ. Chest Radiology: The Essentials. 3rd ed. Philadelphia, PA: Wolters Kluwer Health; 2015.)
GCA accounts for almost one-fifth of cases of FUO in the elderly. Clinical manifestations include headache, jaw claudication, fever, and elevated erythrocyte sedimentation rate. The temporal artery may be tender, thickened, or nodular on examination (see Figure 50-2). Temporal artery biopsy can confirm the diagnosis. Glucocorticoids are the treatment of choice.7 There is no confirmatory diagnostic test for PMR; it is strictly a clinical diagnosis. PMR is closely associated with GCA (it most often occurs in isolation but develops in a significant proportion of patients with GCA). Clinical manifestations include pain and stiffness in the muscles of the neck, shoulders, lower back, hips, and thighs. There is often a dramatic response to glucocorticoid therapy.7 FUO is a rare presentation of IBD and has been reported mostly in patients with ulcerative colitis. Fever is present in almost one-half of patients with ulcerative colitis at the time of presentation. It is often low-grade and may have been present for weeks or longer without having been recognized. Colonoscopy with or without nuclear medicine imaging (eg, labeled leukocyte scintigraphy, positron emission tomography) can be helpful in identifying atypical cases of IBD presenting with FUO. Nuclear medicine imaging is often used to direct the colonoscopic evaluation.16 FMF is an autosomal recessive hereditary disease that predominantly affects ethnic groups near the Mediterranean Sea, particularly Arabs, Armenians, Turks, North Africans, and Jews. Symptoms include periodic attacks of fever and serositis (eg, peritonitis, pleuritis, synovitis) with or without painful erysipelas-like erythema of the lower extremities. The first attack occurs before 10 years of age in most patients, and before 20 years of age in the vast majority of patients. Attacks tend to last 1 to 4 days. Symptom-free intervals are highly variable, even within the same individual, and may be as short as days or as long as years.
439
Colchicine is the treatment of choice. Family history is key to identifying FMF.7,17
440
Malignant Causes of Fever of Unknown Origin
What historical information is important to obtain when considering malignant causes of fever of unknown origin? What is the most common cause of fever of unknown origin in patients with known malignancy?
Information pertinent to malignant causes of FUO includes a history of malignancy and significant weight loss (>2 lbs/wk) with anorexia.14 Infection is responsible for most cases of FUO in patients with known malignancy. The malignancy itself is responsible for fever in just under one- half of cases.18
What are the malignant causes of fever of unknown origin?
Two general types of hematologic malignancies. Often associated with macrocytosis. The biopsy of a hepatic mass stains positive for human epidermal growth factor receptor 2 (HER2). A 63-year-old man with an extensive smoking history presents with intermittent fever and is found to have hematuria and polycythemia. A 64-year-old man with Streptococcus gall- olyticus (formerly Streptococcus bovis) endocarditis. Occurs in patients with cirrhosis. Tumor plop.
Lymphoma and leukemia.
Myelodysplastic syndrome (MDS).
Metastatic breast cancer.
Renal cell carcinoma.
Colon cancer.
Hepatocellular carcinoma (HCC).
Atrial myxoma.
What are the Lymphoma accounts for around one-quarter of the malignant causes of FUO. The presence of lymphadenopathy, characteristics of splenomegaly, and elevated serum lactate dehydrogenase levels can be suggestive. Physical examination, cross-lymphoma sectional imaging of the chest, abdomen, and pelvis, and bone marrow examination can often identify sites of presenting with involvement. However, in patients with FUO, lymphoma may occur exclusively in non-nodal locations (eg, blood fever of vessel lumina [ie, intravascular lymphoma], central nervous system, spleen, liver, bone marrow). Lymphoma unknown origin? presenting with FUO is generally associated with rapid progression and poor prognosis.7,18 Is fever of FUO is more common in patients with acute leukemia (often the nonlymphocytic types). It is rare in patients with unknown origin chronic leukemia, most often occurring after Richter transformation to lymphoma. In patients with acute leukemia more commonly presenting with FUO, peripheral blood smear is frequently unrevealing (ie, aleukemic leukemia) and bone marrow associated with examination is necessary to establish the diagnosis.5,19 acute or chronic leukemia?
441
characteristics of myelodysplastic syndrome presenting with fever of unknown origin?
differentiation of the hematopoietic stem cell, resulting in ineffective hematopoiesis. MDS typically occurs in patients >50 years of age. Most patients are asymptomatic or present with manifestations related to at least 1 peripheral blood cytopenia (anemia, thrombocytopenia, neutropenia). In a minority of cases, fever is the dominant and presenting feature. Peripheral blood smear evaluation may provide diagnostic clues, including macrocytosis (Figure 21-5), anisocytosis, poikilocytosis, nucleated red blood cells, acanthocytosis, hypogranulation and hyposegmentation of the neutrophils, and hypogranularity or hypergranularity and enlargement of the platelets. Bone marrow examination is necessary to establish the diagnosis.5
FIGURE 21-5 Oval macrocytes demonstrated on peripheral blood smear. A red blood cell is typically similar in size to the nucleus of a resting lymphocyte (arrow). The presence of oval macrocytes may indicate megaloblastic anemia or myelodysplastic syndrome. (From Pereira I, George TI, Arber DA. Atlas of Peripheral Blood: The Primary Diagnostic Tool. Philadelphia, PA: Wolters Kluwer Health; 2013.)
What are the Renal cell carcinoma most often presents with weight loss and fatigue, but fever is part of the presentation in up to 15% characteristics of of cases. Microscopic hematuria and peripheral erythrocytosis (secondary to Increased erythropoietin production by renal cell the tumor) may provide clues. Other causes of fever and renal mass include renal abscess, renal tuberculosis, carcinoma xanthogranulomatous pyelonephritis, and renal malakoplakia.5,7,20 presenting with fever of unknown origin? What is the most The liver is the most frequent site of metastasis in patients with metastatic carcinoma presenting with FUO. Other sites frequent site of include the bone, lung, adrenal glands, and abdominal lymph nodes. Cross-sectional imaging and tissue biopsy are metastasis in usually necessary to make the diagnosis.3,18 patients with metastatic carcinoma presenting with fever of unknown origin? How often does Colon cancer infrequently presents with fever, although when it does, fever is due to infection in around one-half of colon cancer cases. Bacteremia from Streptococcus gallolyticus, Escherichia coli, and Clostridium septicum can lead to endocarditis or present with prosthesis infection. Colonoscopy is the diagnostic study of choice.18 fever? Which Although serum α-fetoprotein (AFP) can be elevated in benign liver conditions, it is often found in higher laboratory test concentration in patients with HCC (typically >500 ng/mL), and can be a useful test in patients with FUO who are at can be helpful in risk for HCC.21 patients at risk for hepatocellular carcinoma? How common is Fever is present in around one-third of patients with atrial myxoma. Other manifestations include syncope, heart fever in patients failure, peripheral or pulmonary emboli, weight loss, myalgias, arthralgias, and rash. Some cases are associated with an with atrial extra heart sound (tumor plop), which most often occurs in early diastole. Echocardiography can confirm the myxoma? diagnosis.5,7
442
Other Causes of Fever of Unknown Origin
What are the other causes of fever of unknown origin?
A careful medication review, including over-the-counter and herbal substances, is a key component in the evaluation of FUO. Consider this condition in patients with Virchow’s triad. Often associated with tender hepatomegaly and jaundice. A 32-year-old female nurse presents with complaints of intermittent fever with temperature up to 41°C for several weeks; she appears well on physical examination. A previously healthy 29-year-old man presents with recurrent daily fever several weeks after sustaining abdominal trauma and is found to have anemia. A young woman who recently gave birth complains of anterior neck discomfort, heat intolerance, and palpitations.
What are the characteristics of drug fever?
How common is fever in patients with pulmonary embolism?
Which laboratory feature is helpful in identifying alcoholic hepatitis?
What are the main clues to the presence of factitious fever?
What are the characteristics of hematoma presenting with fever of unknown origin?
What are the endocrinologic causes of fever of unknown origin?
Drug fever.
Venous thromboembolism (ie, deep vein thrombosis, pulmonary embolism [PE]). Alcoholic hepatitis. Factitious disorder.
Intra-abdominal hematoma.
Postpartum thyroiditis.
Numerous medications can cause fever, including some that are used to treat fever (eg, nonsteroidal anti-inflammatory drugs [NSAIDs], antibiotics). Fever usually occurs several weeks after initiating the medication but can present at any point. Peripheral eosinophilia or skin rash are present in around one-fifth of patients, and can be important clues to the diagnosis. Fever usually resolves within 2 days of stopping the causative agent.5,7 Fever occurs in around one-half of patients with acute PE. Temperature is typically <39°C, and fever resolves a few days after therapy is initiated. However, fever can persist for weeks, usually when recurrent emboli go undetected and untreated. Pulmonary symptoms can be absent or unimpressive in some cases, leading to a protracted course.5 Alcoholic patients may be hesitant to disclose a history of alcohol use. Confirming the presence of ethanol in serum or urine samples may be helpful; however, alcohol consumption is often stopped days or weeks before patients present with alcoholic hepatitis. Leukocytosis is frequently present, but this is nonspecific, particularly in patients with FUO. A more specific feature is a mild to moderate elevation of aminotransferases, with an AST:ALT ratio >2:1.5 Factitious fever most often occurs in young women who work or have previously worked in health care. Patients often appear well on examination; however, there may be signs of self-mutilation or injection. Other clues include markedly elevated temperatures (>41°C) with the absence of associated tachycardia or diaphoresis, and rapid defervescence.5 Hematoma can cause fever when it occurs in an enclosed space, usually the abdominal cavity or retroperitoneal space. It can also occur in patients with aortic dissection, usually preceded by a transient episode of chest, back, or abdominal pain. Cross-sectional imaging is a key diagnostic modality to identify the presence of a hematoma. In some cases, fever is caused by secondary infection.5,7 Thyrotoxicosis is the most frequent endocrinologic cause of FUO. Other endocrinologic causes of FUO include adrenal insufficiency and pheochromocytoma.5
443
444 Case Summary of intermittent fever, migratory inflammatory arthritis, and painless e A 37-year-old Syrian man presents with a prolonged clinical cours oral ulcers following a diarrheal illness.
What is the most likely diagnosis in this patient? Reactive arthritis.
445
Bonus Questions
What is reactive Reactive arthritis isione of the seronegative spondyloarthritides and is characterized by a sterile, asymmetric inflammatoryimono- or oligoarthritisithat develops 1 tohe arthritis? 4 weeks following infection elsewhere in the body (usually Gastrointestinal or genitourinary infections). The inciting infect on can be asymptomat c in some cases. T classic triad of post nfectious arthritis, conjunctivitis, and nongonococcal urethritis (ie, Reiter’s syndrome) represents a subtype of reactive arthritis. In many cases, postinfectious arthritis occurs in isolation or with only 1 of the other 2 extra-articular features. Other extra-articular manifestations of reactive arthritis include tendinitis, bursitis, anterior uveitis, erythema nodosum, and circinate balanitis.15,22 Reactiveis most Organisms most often associated with reactive arthritis include Chlamydia trachomatis, Yersinia species, Salmonella enterica, Campylobacter species, Shigella species, and likely to develop Clostridium difficile Escherichia coli Chlamydia pneumoniae arthritis . It can also develop following infection with other organisms, such as and . 15,22,23 after infections with what organisms? What is the y of Reactive arthritis occurs worldwide, with an annual incidence in some populations of up to 30fper 100,000. It occurs most frequently in young adults 20tto 40 years of reactive Chlamydia trachomatis epidemiolog age. Compared with women, men are more likely to develop and experience a severe course o reactive arthritis related to urethri is. 15,22 arthritis? suggestive ofs in manifestations, including enthesitis, conjunctivitis, urethritis, oral ulcers,tand theiabsenceioftserologic evidence of RA are also consistentiwith the diagnosis.iThe patient significance of be unusual for the first attack to occur after 20 years of age, and there is no family history of FMF in this patient. Genetic testing tolevaluate for FMF may be helpful ind What feature In this case, the development of migratory, asymmetric, sterile inflamma ory oligoarthritis affecting the larger joints of the lower extrem ties following a this case are Gastrointestinal illness is characteristic of reactive arthritis. As a young adult, the patient n his case fits the epidemiologic profile. The presence of extra-art cular arthritis? reactive in this case should undergo an ophthalmologic examination to evaluate for uveit s. 22 What is the The Syrian ancestry of the patient in this case should bring FMF and Behçet’s disease into consideration. Although FMF is relative y more common in Syrians, it woul the patient’s this case. Behçet’s disease is also known as “Silk Road disease” and should be considered in any patient of Syrian descent who presents with FUO, particularly given Syrian ancestry the presence of oral ulcers. However, the oral ulcers of Behçet’s disease are typically painful. The pathergy test to evaluate for Behçet’s disease may be helpful in this in this case? case. 17,24 What is the then The spondyloarthritides, including reactive arthritis and ankylosing spondylitis (present in a first-degree relative in this case), are associated with human leukocyte What is the Cases of reactive arthritis in which there is ongoing infection (eg, Chlamydia trachomatis urethritis) require treatment with antibiotics. NSAIDs are first-line treatment for family history i this case? relevance of antigen B27 (HLA-B27). HLA-B27 positive patients tend to experience more severe arthritis with a protracted course. 22 treatment for arthritis. Some patients may also benefit from intra-articular glucocorticoids. Patients who do not respond to NSAIDs may benefit from systemic glucocorticoids. reactive ? Disease-modifying antirheumatic drugs (eg, sulfasalazine) may be necessary in some cases. Patients with extra-articular manifestations may benefit from additional What is thef Most patients with reactive arthritis fully recover within 2 to 6 months. However, up to one-fifth of patients experience chronic disease, defined by the persistence of arthritis therapy (eg, topical glucocorticoids for uveitis). 15,22 reactive prognosis o symptoms beyond 6 months. Some of these patients may later develop features of other spondyloarthritides. 15 arthritis?
446 Key Points
measured temperaturel>38.3°C on several occasions, and failure to infectious, noninfectious inflammatory, malignant, and other.ries: geographic factors. In the modern industrialized world, infections 25%; malignancy, 15%; and miscellaneous or idiopathic causes History and physical examination are critically important for FUO is defined as an il ness lasting longer than 3 weeks, with reach a diagnosis despite thorough investigation. The causes of FUO can be separated into the following catego Relative prevalence of the causes of FUO depends on host and account for 25% of cases; noninfectious inflammatory conditions, make up the remaining cases. directing the diagnostic workup in patients with FUO. Diagnostic studies that may be helpful in identifying the cause of
FUO include varioustlaboratory studies (eg, serologies), imaging studies (eg, cross-sec ional imaging), endoscopic studies (eg, colonoscopy), and histopathologic studies (eg, lymph node biopsy). Treatment and prognosis of FUO depend on the underlying cause
Prognosis is excellent in patients with idiopathic FUO but poor in . those with malignant FUO.
447
References 1. Longo DL, Fauci AS, Kasper DL, Hauser SL, Jameson JL, Loscalzo J, eds. Harrison’s Principles of Internal Medicine. 18th ed. New York, NY: McGraw-Hill; 2012.
-
Hayakawa K, Ramasamy B, Chandrasekar PH. Fever of unknown origin: an evidence- based review. Am J Med Sci. 2012;344(4):307-316.
-
Petersdorf RG, Beeson PB. Fever of unexplained origin: report on 100 cases. Medicine (Baltim). 1961;40:1-30.
-
Kazanjian PH. Fever ofiunknown origin: review of 86 patients treated in community 5. Hirschmann JV. Fever of unknown origin in adults. Clin Infect Dis. 1997;24(3):291-300; hospitals. Clin Infect D s. 1992;15(6):968-973. quiz 1-2.
-
Mourad O, Palda V, Detsky AS. A comprehensive evidence-based approach to fever of unknown origin. Arch Intern Med. 2003;163(5):545-551.
-
Arnow PM, Flaherty JP. Fever of unknown origin. Lancet. 1997;350(9077):575-580.
-
pulmonary tuberculosis,in patients with AIDS: spectrum of radiographic findings Greenberg SD, Frager D Suster B, Walker S, Stavropoulos C, Rothpearl A. Active (including a normal appearance). Radiology. 1994;193(1):115-119.
-
Baddour LM, Wilson WR, Bayer AS, et al. Infective endocarditis in adults: diagnosis, healthcare professionals from the American Heart Association. Circulation. antimicrobial therapy, and management of complications:a scientific statement for 2015;132(15):1435-1486.
-
Alavi SM, Nadimi M,)ZamanitGA. Changing pattern of infectious etiology of fever of unknown origin (FUO in adul patients in Ahvaz, Iran. Caspian J Intern Med. 2013;4(3):722-726.
-
Bleeker-Rovers CP, Vos FJ, de Kleijn EM, et al. A prospective multicenter study on fever of unknown origin: the yield of a structured diagnostic protocol. Medicine.
2007;86(1):26-38. 12. Tsukahara M, Tsuneoka H, Iino H, Murano I, Takahashi H,lUchida M. Bartonella henselae infection as a cause of fever of unknown origin. J C in Microbiol. 2000;38(5):1990-1991. . Gopalarathinam R, Orlowsky E, Kesavalu R, Yelaminchili S. Adult onse Still’s
13disease: a:review on diagnostic workup and treatment options. Case ReptRheumatol. 14. Cunha BA, Lortholary O, Cunha CB. Fever of unknown origin: a clinical approach. 2016;2016 6502373. Am J Med. 2015;128(10):1138 e1-1138 e15.
-
Kim PS, Klausmeier TL, Orr DP. Reactive arthritis: a review. J Adolesc Health. 2009;44(4):309-315.
-
Voukelatou P, Sfendouraki E, KarianostT,let al.;Ulcerative colitis activity presenting as fever of unknown origin, in a patient wi h ongstanding disease under continuous treatment with mesalazine. Case Rep Med. 2016 2016:4396256.
-
Sari I, Birlik M, Kasifoglu T. Familial Mediterranean fever: an updated review. Eur J Rheumatol. 2014;1(1):21-33.
-
Loizidou A, AounlM, Klastersky J. Fever of unknown origin in cancer patients. Crit 19leukemia versus lymphoma (Richter’s transformation). Heart Lung. 2005;34(6):437-441. Rev Oncol Hemato . 2016;101:125-130. . Cunha BA, Mohan S, Parchuri S. Fever of unknown origin: chronic lymphatic
448
-
Chandrankunnel J, CunhaiBA, Petelin A, Katz D. Fever;of unknown origin (FUO) and 21. Wu JT. Serum alpha-fetoprotein and its lectin reactivity in liver diseases: a review. a renal mass: renal cell carc noma, renal tuberculosis, renal malakoplakia, or xanthogranulomatous pyelonephritis? Heart Lung. 2012 41(6):606-609. Ann Clin Lab Sci. 1990;20(2):98-105.
-
Hannu T. Reactive arthritis. Best Pract Res Clin Rheumatol. 2011;25(3):347-357.
- Townes JM. Reactive arthritis after enteric-infections in the United States: the problem 24. Sakane T, Takeno M, Suzuki N, Inaba G. Behcet’s disease. N Engl J Med. of definition. Clin Infect Dis. 2010;50(2):247 254. 1999;341(17):1284-1291.
449
CHAPTER 22
450
Hypotension
451 Case: A 48-year-old man with cool extremities A 48-year-old man with a history of hypertension and insulin-
dependent type 2 diabetes mellitus is admitted to the hospital with The pain is located in the center of the chest with radiation to the jaw. but is now constant. He is also short of breath and feels light-headed. alcohol use. His father died of a myocardial infarction (MI) at 52 years episodes of increasing chest discomfort over the course of a few days. It is pressure-like in quality. It had been intermittent for a few days He smokes 1 pack of cigarettes per day but denies any significant of age. Heart rate is 144 beats per minute, blood pressure is 70/40 mm Hg
(similar in all extremities), respiratory rate is 36 breaths per minute, and hemoglobin oxygen saturation by pulse oximetry is 88% on 4 L
supplemental oxygen. Jugular venous pressure (JVP) is 16 cm H O. An 2 extra heart sound is heard just af er S2 with the bell of the stethoscope over the apex. There are bilateraltlate inspiratory rales. The extremities are cool to the touch, and the peripheral pulses are weak. Electrocardiogram (ECG) is shown in Figure 22-1.
wChest radiograph shows bilateral patchy ground glass opacities ith bilateral pleural effusions.
FIGURE 22-1 (From Woods SL, Froelicher ES, Motzer SA, Bridges EJ. Cardiac Nursing. 6th ed. Philadelphia, PA: Wolters Kluwer Health; 2010.)
What is the most likely cause of hypotension in this patient?
What is the relationship between arterial blood pressure, cardiac output (CO), and systemic vascular resistance (SVR)? What are the main determinants of cardiac output? How is blood pressure regulated?
Mean arterial pressure (MAP) is the product of cardiac output and systemic vascular resistance.1MAP =CO ×SVR
Cardiac output is equal to the forward stroke volume (SV) of the left ventricle per beat multiplied by heart rate (HR).CO =SV ×HRIn the setting of hypotension, neurally-mediated compensatory increases in heart rate and stroke volume will occur.2 Moment-to-moment control of blood pressure is regulated by the neurally- mediated baroreceptors found in the carotid sinus and aortic arch. Long-term control of blood pressure is primarily regulated via the hormonally- mediated renin-angiotensin-aldosterone system (Figure 22-2).3
452
FIGURE 22-2 Mechanisms of blood pressure regulation. Dashed lines represent stimulation of blood pressure
How is blood pressure measured?
What conditions optimize the indirect measurement of blood pressure?
What are the symptoms of hypotension? What are the physical findings of hypotension? What is hypotensive shock?
What are the 4 general mechanisms of hypotension?
What is the relative prevalence of each category of hypotension in patients with shock?
regulation. Solid lines represent response to stimulation of kidneys and baroreceptors. (From Porth CM. Essentials of Pathophysiology: Concepts of Altered Health States. Philadelphia, PA: Lippincott Williams & Wilkins; 2003.)
Blood pressure is usually measured indirectly by using a stethoscope and a sphygmomanometer. It can be measured directly by cannulating a peripheral artery, which is the method of choice in patients with significant hemodynamic compromise.1 Korotkoff sounds, used to measure blood pressure, are low pitched and are best appreciated in a quiet room using the bell of the stethoscope. The size of the bladder on the sphygmomanometer can affect the accuracy of blood pressure measurements. The length of the bladder should be ≥80% of the circumference of the upper arm, and the width should be ≥40% of the circumference of the upper arm. Bladders that are too large result in falsely low blood pressure readings, whereas bladders that are too small result in falsely elevated blood pressure readings.1 Symptoms of hypotension may include light-headedness, dizziness on sitting up or standing (ie, orthostasis), syncope, dyspnea, blurry vision, malaise, and confusion. In addition to low arterial blood pressure, physical findings of hypotension may include positive orthostatic vital signs, cool or warm extremities (depending on the underlying cause), and diminished peripheral pulses. Hypotensive shock occurs when tissue oxygenation is inadequate for the demands of aerobic metabolism. In adults, it is typically associated with acute systolic arterial pressure <90 mm Hg or MAP <70 mm Hg (these thresholds may be higher in patients with chronic hypertension). Clinical manifestations include cutaneous changes (eg, cool, clammy, cyanotic skin), Decreased urine output, and delirium. Lactate levels in blood are elevated due to Increased generation of lactate from a cellular shift to anaerobic metabolism, and Decreased clearance related to impaired liver function.1,4 Hypotension can be hypovolemic, cardiogenic, distributive, or obstructive.
Among patients with shock in the industrialized world, distributive hypotension causes approximately 65% of cases (among which sepsis is by far the most frequent etiology). Hypovolemic hypotension and cardiogenic hypotension account for around 15% of cases each, and <5% are due to obstructive hypotension.4
453
Hypovolemic Hypotension
What are the fundamental pathophysiologic mechanisms of low blood pressure in hypovolemic hypotension? In the setting of hypovolemic shock, are the extremities typically warm or cool to the touch? Which of the following patterns of jugular venous pressure, cardiac output, and systemic vascular resistance are characteristic of hypovolemic hypotension?
A B C Why is it important to use isotonic fluid when resuscitating patients with hypovolemic hypotension?
Hypovolemic hypotension occurs as a result of Decreased hydrostatic pressure within blood vessels, and Decreased cardiac preload with an associated decrease in cardiac output.
Hypovolemic shock is typically associated with cool extremities. This reflects the compensatory increase in SVR that aims to shunt blood to the vital organs.
Hypovolemic hypotension is characterized by Decreased JVP (primary issue), Decreased CO, and Increased SVR (answer C).1
JVP CO SVR ↑ ↓ ↑ ↓ ↑ ↓ ↓ ↓ ↑ Isotonic solutions (eg, crystalloids, colloids) are more effective at expanding intravascular fluid volume compared with hypotonic solutions. The infusion of isotonic fluid does not generate a tonicity gradient between the extracellular and intracellular fluid compartments. Therefore, the infusate will remain within the extracellular compartment; one-fourth of which consists of the intravascular space. In contrast, hypotonic solutions generate a tonicity gradient favoring redistribution of infusate into the intracellular compartment. For example, if 1 L of free water is infused, only one-third of it will remain in the extracellular compartment and only one-twelfth in the intravascular space (Figure 22-3).
FIGURE 22-3 Approximate relationships of body water compartments to total body weight. Total body water (TBW) is approximately 60% of body weight (BW) in the average 70-kg man. The majority of water is distributed within the intracellular compartment, whereas a relatively small amount is distributed within the intravascular space. (From Rennke HG, Denker BM. Renal Pathophysiology: The Essentials. 4th ed. Philadelphia, PA: Lippincott Williams & Wilkins; 2014.)
What are the causes of hypovolemic hypotension?
Common in elderly patients who live alone. Often encountered in trauma patients. Polyuria. True isotonic extracellular fluid loss. Massive fluid loss through the skin.
Poor oral intake. Hemorrhagic shock. Renal salt wasting (eg, diuretic use). Gastrointestinal losses (eg, diarrhea, vomiting). Severe burn injury.
454
What are the causes of poor oral intake? Which areas of the body can hide large amounts of blood?
Which laboratory test indicates that volume depletion may be related to renal salt wasting? What is the significance of true isotonic extracellular fluid loss? Which validated formula is useful for guiding fluid replacement in the setting of severe burns?
Common causes of poor oral intake include lack of access, anorexia nervosa, loss of appetite, early satiety, intestinal obstruction, socioeconomic factors, delirium, mental health disorders, and sitophobia from chronic mesenteric ischemia. In some cases, the patient readily provides a history of poor oral intake, but in other cases, it must be inferred or gathered from family members. Extensive bleeding can occur within the retroperitoneal space, pelvis, or thigh without significant physical findings. A history of trauma, predisposing factors (eg, anticoagulation use, liver disease), or the presence of anemia may provide clues. Acute blood loss may not initially result in Decreased hemoglobin concentration. Imaging studies are key to making the diagnosis. In patients with hemorrhagic shock, resuscitation with crystalloid solutions or packed red blood cells (RBC) can worsen or induce coagulopathy because of a dilutional effect on coagulation proteins and platelets. To prevent this, massive resuscitation should include the administration of fresh frozen plasma and platelets (aiming for a 1:1:1 ratio of plasma:platelets:RBCs as a general rule of thumb). Metabolic alkalosis and hypocalcemia can also develop in these patients as a result of the citrate present in blood products.5,6 Extrarenal causes of volume depletion are associated with urine sodium concentration <20 mEq/L. In contrast, volume depletion related to renal salt wasting (eg, primary adrenal insufficiency, diuretic use) tends to be associated with relatively higher urine sodium concentrations (>20-30 mEq/L).7-9
Loss of extracellular fluid that is hypotonic relative to intracellular fluid (eg, pure water) will generate a tonicity gradient between the extracellular and intracellular compartments, which favors the movement of water into the extracellular compartment, thereby restoring some extracellular fluid volume. In contrast, loss of extracellular fluid that is isotonic relative to intracellular fluid (eg, Gastrointestinal loss) does not generate a tonicity gradient between the extracellular and intracellular compartments, and there is no resultant restoration of extracellular fluid volume from the intracellular space. Therefore, extracellular isotonic fluid loss tends to be associated with more severe clinical manifestations.10 The Parkland formula can guide fluid replacement in burn victims, using the patient’s body weight (in kg) and the estimated percentage of body surface area burned. During the initial 24 hours, crystalloid should be given at a rate of 4 mL/kg/% burn (for adults).11
455
Cardiogenic Hypotension
What is the fundamental pathophysiologic mechanism of low blood pressure in cardiogenic hypotension? In the setting of cardiogenic shock, are the extremities typically warm or cool to the touch?
Which of the following patterns of jugular venous pressure, cardiac output, and systemic vascular resistance is characteristic of cardiogenic shock?
A B C
Cardiogenic hypotension occurs as a result of pump failure with an associated decrease in cardiac output.
Cardiogenic shock is typically associated with cool extremities. This reflects the compensatory increase in SVR that aims to shunt blood to the vital organs. However, when MI is the cause of cardiogenic shock, there is a subset of patients who develop systemic inflammatory response syndrome (SIRS), causing Decreased SVR and warm extremities.12 Cardiogenic shock is characterized by Increased JVP, Decreased CO (primary issue), and Increased SVR (answer A).1
JVP CO SVR ↑ ↓ ↑ ↓ ↑ ↓ ↓ ↓ ↑
What are the causes of cardiogenic hypotension?
Diseases of heart muscle that generally result in concentric or eccentric hypertrophy. Sometimes associated with palpitations. A 52-year-old man with an extensive smoking history presents with chest pain and hypotension and is found to have 2 mm ST-segment elevation in leads II, III, and aVF on ECG. A 27-year-old man with Marfan syndrome presents with syncope and is found to have hypotension with a wide pulse pressure, a III/IV decrescendo diastolic murmur, de Musset sign (to-and-fro head bob), and Traube sign (a “pistol-shot” sound heard with the stethoscope over the femoral artery). Sudden development of pulmonary hypertension. Functionally similar to aortic stenosis, but the valve itself may not be the culprit. A previously healthy 33-year-old woman presents with months of progressive dyspnea on exertion and is found to have elevated JVP, a right ventricular heave, and a loud pulmonic component of the second heart sound.
What class of medications can be used to directly increase cardiac output in patients with cardiogenic shock?
What is the immediate treatment of choice for atrial fibrillation
Cardiomyopathy (see chapter 4, Heart Failure).
Dysrhythmia. Acute inferior ST-elevation myocardial infarction.
Acute aortic regurgitation.
Acute pulmonary embolism (PE). Left ventricular outflow tract obstruction.
Primary pulmonary hypertension.
Inotropic agents (eg, dobutamine) can increase myocardial contractility and are useful in the management of cardiogenic shock. Additional support with vasopressor agents (eg, norepinephrine) may also be helpful. Mechanical support with intra-aortic balloon counterpulsation (ie, intra-aortic balloon pump) may be necessary to maintain coronary and peripheral perfusion in select cases of cardiogenic shock. Intra-aortic balloon counterpulsation increases diastolic pressure via diastolic balloon inflation, and augments left ventricular performance by decreasing afterload via systolic balloon deflation.13 Atrial fibrillation with hemodynamic compromise (eg, hypotension or decompensated heart failure) should be treated with immediate synchronized cardioversion using direct current (DC) electric
456
associated with hypotension? shocks. Patients should be premedicated with sedatives and narcotics when possible, as electrical cardioversion is traumatic and painful.1 What additional therapies should In addition to hemodynamic support with inotropes and vasopressors, pharmacologic management of be used in patients with cardiogenic shock related to acute MI includes antiplatelet and antithrombotic therapy. Survival in cardiogenic shock related to acute patients with cardiogenic shock related to MI is improved with early reperfusion, particularly myocardial infarction? revascularization, whether percutaneous (ie, percutaneous coronary intervention) or surgical (ie, coronary artery bypass grafting). Thrombolytic therapy, although less effective than revascularization, should be given when revascularization is unavailable or significantly delayed. Mechanical support with an intra-aortic balloon pump can be helpful in select patients. In some cases, left ventricular assist devices or extracorporeal life support should be considered.13 Which valvulopathies are Cardiogenic shock can occur as a result of acute-onset severe aortic or mitral regurgitation. Because associated with cardiogenic left ventricular adaptation has not had time to develop, the hemodynamic consequences of acute shock? regurgitant valvular lesions are more severe than those of chronic valvular regurgitation. Causes of acute-onset aortic regurgitation include Infective endocarditis with leaflet perforation, type A aortic dissection, blunt trauma, prosthetic valve dysfunction, and iatrogenic injury (eg, valve injury during cardiac catheterization). Causes of acute-onset mitral regurgitation include Infective endocarditis with leaflet perforation, chordal rupture related to myxomatous degeneration, papillary muscle rupture from MI, acute cardiomyopathy (ie, functional mitral regurgitation), acute rheumatic fever, prosthetic valve dysfunction, and iatrogenic injury.14 What is the treatment for acute Patients with acute PE that are hypotensive should receive support with intravenous fluids and pulmonary embolism with vasopressors. In patients without a contraindication, systemic thrombolytic therapy should be hemodynamic compromise? administered. Embolectomy (either surgical or catheter-based) may be considered when there is contraindication to, or failure of, thrombolytics.15 Where are the 3 general sites of The general sites of left ventricular outflow tract obstruction are subvalvular (eg, hypertrophic left ventricular outflow tract obstructive cardiomyopathy), valvular (eg, aortic stenosis), and supravalvular (eg, coarctation of obstruction? aorta). Coarctation of the aorta typically results in hypertension of the upper extremities, with normal or low blood pressure in the lower extremities. What physical findings are The physical findings of pulmonary hypertension include sinus tachycardia, elevated JVP, right associated with pulmonary ventricular heave, right ventricular gallop, loud pulmonic component of the second heart sound (P2), hypertension? ascites, and peripheral edema. The presence of clear lungs can help differentiate right-sided heart failure from acute left-sided heart failure.16
457
Distributive Hypotension
What is the fundamental pathophysiologic mechanism of low blood pressure in distributive hypotension? In the setting of distributive shock, are the extremities usually warm or cool to the touch? Which of the following patterns of jugular venous pressure, cardiac output, and systemic vascular resistance is characteristic of distributive shock?
A B C
Distributive hypotension occurs as a result of pathologic peripheral vasodilation with an associated decrease in SVR. Distributive shock is typically associated with warm extremities, a reflection of the decrease in SVR that is characteristic of this condition. Distributive shock is characterized by Decreased JVP, Increased CO, and Decreased SVR (primary issue) (answer B).1 JVP CO SVR ↑ ↓ ↑ ↓ ↑ ↓ ↓ ↓ ↑
What are the causes of distributive hypotension?
A 55-year-old woman presents with dyspnea and purulent cough and is found to have a temperature of 39.2°C, heart rate of 132 beats per minute, blood pressure of 88/55 mm Hg, and leukocytosis of 18 K/µL. A 42-year-old man with a history of alcohol abuse presents with nausea and postprandial epigastric abdominal pain that radiates to the back. A 43-year-old man with a history of hypertension and poor medication adherence is admitted to the hospital for an elective procedure and unexpectedly becomes hypotensive before the procedure. Hypotension accompanied by bronchoconstriction and an urticarial eruption. Hyperpigmentation, hyponatremia, and hyperkalemia. Sudden loss of autonomic tone as a result of spinal cord injury. A 76-year-old man with a history of Parkinson’s disease presents with months of light-headedness and is found to have a blood pressure of 188/105 mm Hg supine that drops to 90/50 mm Hg on standing, with no appreciable change in heart rate.
What are the general principles of treating septic shock?
What is the underlying mechanism of hypotension in patients with acute pancreatitis?
Sepsis caused by pneumonia.
Acute pancreatitis.
Medication. Patients with poor medication adherence are often mistakenly diagnosed with refractory hypertension and prescribed unnecessary antihypertensive medications. These patients are at risk for becoming hypotensive in the hospital setting when home medications are reliably administered.
Anaphylaxis.
Primary adrenal insufficiency.
Neurogenic shock. Dysautonomia.
Sepsis accounts for the vast majority of cases of distributive shock. The principles of treatment include administering supplemental oxygen to increase oxygen delivery to the tissues (mechanical ventilation may be necessary), isotonic intravenous fluids (eg, crystalloid), broad-spectrum antibiotics, and vasopressors if necessary. The placement of an arterial catheter to monitor arterial blood pressure and sample blood, and the insertion of a central venous catheter to monitor hemodynamics and administer fluids and vasoactive medications, are important steps in the management of septic shock.4 The mechanism of hypotension in patients with acute pancreatitis is incompletely understood but is related in part to a systemic inflammatory response syndrome that causes vasodilation. Up to one-fifth of patients with acute pancreatitis develop severe disease with systemic inflammatory response. Hypotension is associated with a higher risk of death. Aggressive fluid resuscitation is an important
458
What medications can cause hypotension?
What is the initial pharmacologic treatment for anaphylaxis?
What is the initial treatment for adrenal crisis?
What is neurogenic shock?
What are the general principles of treatment for hypotension associated with dysautonomia?
part of management.17 Medications most commonly associated with hypotension (particularly orthostatic hypotension) include diuretics, sedatives, centrally acting adrenergic blockers, peripherally acting adrenergic blockers, vasodilators, β-blockers, and nitrates.18 The cornerstone of pharmacologic management of anaphylaxis is the administration of epinephrine (generally at a dose between 0.3-0.5 mg in adults, depending on weight). Intramuscular injection is the preferred route, and the optimal site of injection is the vastus lateralis muscle. Epinephrine can be lifesaving for patients with anaphylaxis, and there is no alternative agent. Adjunctive medications include antihistamines, glucocorticoids, and bronchodilators.19 Adrenal crisis in adult patients should be treated with fluid resuscitation and parenteral hydrocortisone. An initial injection of a 100 mg bolus of hydrocortisone should be followed by 200 mg daily (eg, 50 mg every 6 hours). If hydrocortisone is unavailable, prednisolone can be used instead. Identifying and addressing the underlying trigger of adrenal crisis is also important for management.20 Neurogenic shock describes the development of hypotension following disruption of sympathetic pathways in patients with severe spinal cord lesions at the level of T-6 or higher. Intravenous fluids and vasopressor support to achieve a minimum MAP of 85 mm Hg may be helpful in preventing secondary ischemic injury to the spinal cord. However, vasopressors must be used cautiously in trauma patients because of the risk of increasing intracranial pressure. Bradycardia is often present in patients with neurogenic shock, particularly when the higher levels of the cervical cord are involved. Pharmacologic management (eg, atropine) and temporary pacing may be necessary in these patients to improve hemodynamics.21 Hypotension related to dysautonomia can be challenging to treat. Nonpharmacologic strategies include avoiding sudden postural changes, increasing daily salt and water intake, and using compression stockings. Pharmacologic agents can be used to increase blood volume (eg, fludrocortisone) and peripheral vasoconstriction (eg, midodrine).22
459
Obstructive Hypotension
What is the fundamental mechanism of low blood pressure in obstructive hypotension? In the setting of obstructive shock, are the extremities typically warm or cool to the touch? Which of the following patterns of jugular venous pressure, cardiac output, and systemic vascular resistance is characteristic of obstructive shock?
A B C
Obstructive hypotension occurs as a result of Decreased cardiac filling with an associated decrease in both preload and cardiac output. Obstructive shock is typically associated with cool extremities. This reflects the compensatory increase in SVR that aims to shunt blood to the vital organs. Obstructive shock is characterized by Increased JVP (primary issue), Decreased CO, and Increased SVR (answer A). JVP CO SVR ↑ ↓ ↑ ↓ ↑ ↓ ↓ ↓ ↑
What are the causes of obstructive hypotension?
460
461 Case Summary A 48-year-old man presents with chest pain and severe hypotension, inspiratory rales on auscultation of the lungs, cold extremities, and an and is found to have elevated JVP, an extra heart sound, diffuse abnormal ECG.
What is the most likely cause of hypotension in this patient? Cardiogenic shock.
462
Bonus Questions
What is the most likely ck Acute MI is the most common cause of cardiogenic shock in the industrialized world. An acute MI is likely in this case given the electrocardiographic in this case? 2-6 see Figure 22-1 cause of cardiogenic sho presence of ST elevation in anterolateral leads I, aVL, and V ( ) in the setting of a compatible clinical syndrome. 13 What is the extra heart Heart sounds that occur near S2 include split S2, S3 gallop, opening snap, pericardialtknock, and tumor plop. The extra sound in this case isimost likely an S3 sound in this case? gallop based on location, pitch, and clinical history. S3 gallops are common in the set ing of cardiogenic shock (S4 gallops are also common n the context of acute ischemia). 16,24 Which physical finding in The presence of elevated JVP in this case rules out hypovolemia. this case helps rule out hypovolemic
hypotension? Which physical findings Elevated JVP before fluid resuscitation would be an unexpected finding in patients withidistributive shock. Furthermore, the presence of cool extremities in distributive hypotension in this case make this case indicates Increased SVR, which is discordant with the pathophysiology of distr butive shock. unlikely? Which physical finding in The bilateral inspiratory rales in this case most likely represent acute cardiogenic pulmonary edema from left-sided heart failure. Patients with obstructive obstructive hypotension elevated. this case makes hypotension have signs of right-sided heart failure (eg, elevated JVP), but the lungs are typically clear because left-sided cardiac filling pressures are not unlikely? What general treatment The patient in this case should be treated with ino ropic and vasopressor support as needed and evaluated for an intra-aortic balloon pump. For the acute strategies should be used antiplatelet and antithrombotic therapy should betprovided. Revascularization via percutaneous coronary intervention or coronary artery bypass graftingMI, in this case? should be pursued as soon as possible. If revascularization is not available, fibrinolytic therapy should be given instead. 13
463 Key Points
Symptoms of hypotension include light-headedness, orthostasis, syncope, dyspnea, blurry vision, malaise, and confusion. Physical findings of hypotension include low arterial blood
pressure, positive orthostatic vital signs, cool or warm extremities (depending on the underlying cause), and diminished peripheral pulses.
Hypotensive shock occurs when tissue oxygenation is inadequate Hypotension can be hypovolemic, cardiogenic, distributive, or Hypovolemic hypotension is characterized by Decreased JVP Cardiogenic hypotension is characterized by Increased JVP, Distributive hypotension is characterized by Decreased JVP, Obstructive hypotension is characterized by Increased JVP for the demands of aerobic metabolism. obstructive. (primary issue), Decreased CO, and Increased SVR. Decreased CO (primary issue), and Increased SVR. Increased CO, and Decreased SVR (primary issue). (primary issue), Decreased CO, and Increased SVR.
464
References 1. Marino PL. The ICU Book.;3rd ed. Philadelphia, PA: Lippincott Williams & Wilkins—A 2. Mangrum JM, DiMarco JP. The evaluation and management of bradycardia. N Engl J Wolters Kluwer Business 2007. Med. 2000;342(10):703-709.
- Berne RML, Levy MN. Physiology. 4th ed. St. Louis, MO: Mosby, Inc.; 1998.
- Vincent JL, De Backer D. Circulatory shock. N Engl J Med. 2013;369(18):1726-1734.
-
Gutierrez G, Reines HD, Wulf-Gutierrez ME. Clinical review: hemorrhagic shock. Crit Care. 2004;8(5):373-381. Holcomb JB, Wade CE, Michalek JE, et al. Increased plasma and platelet to red blood
-
cell ratios improves outcome in 466 massively transfused civilian trauma patients. Ann Surg. 2008;248(3):447-458.
-
Chung HM, Kluge R, Schrier RW, Anderson RJ. Clinical assessment of extracellular fluid volume in hyponatremia. Am J Med. 1987;83(5):905-908.
-
Schrier RW. Body water homeostasis: clinical disorders of urinary dilution and concentration. J Am Soc Nephrol. 2006;17(7):1820-1832.
-
Longo DL, Fauci AS, Kasper DL, Hauser SL, Jameson JL, Loscalzo J, eds. Harrison’s Principles of Internal Medicine. 18th ed. New York, NY: McGraw-Hill; 2012.
-
Bhave G, Neilson EG. Volume depletion versus dehydration:-how understanding the 11. Baxter CR, Shires T. Physiological(response to crystalloid resuscitation of severe difference can guide therapy. Am J Kidney Dis. 2011;58(2):302 309. burns. Ann N Y Acad Sci. 1968;150 3):874-894. . Kohsaka S, Menon V, Lowe AM, et al. Systemic inflammatory response syndrome 12after;acute myocardial infarction complicated by cardiogenic shock. Arch Intern Med. 2005 165(14):1643-1650.
-
Reynolds HR, Hochman JS. Cardiogenic shock: current concepts and improving outcomes. Circulation. 2008;117(5):686-697.
-
Mokadam NA, Stout KK, Verrier ED. Management of acute regurgitation in left-sided cardiac valves. Tex Heart Inst J. 2011;38(1):9-19.
-
Torbicki A, Perrier A, Konstantinides S, et al. Guidelines on the diagnosis and management of acute pulmonary embolism of the european society of cardiology management of acute pulmonary embolism: the task force for the diagnosis and (ESC). Eur Heart J. 2008;29(18):2276-2315.
-
Alpert JS, Becker RC. Cardiogenic shock: elements of etiology, diagnosis, and therapy. Clin Cardiol. 1993;16(3):182-190.
-
Garcia M, CalvolJJ. Cardiocirculatory pathophysiological mechanisms in severe acute 18. Sathyapalan T, Aye MM, Atkin SL. Postural hypotension. BMJ. 2011;342:d3128. pancreatitis. Wor d J Gastrointest Pharmacol Ther. 2010;1(1):9-14.
- Dhami S, Panesar SS, Roberts G, et al. Management of anaphylaxis: a systematic review. Allergy. 2014;69(2):168-175 . Bornstein SR, Allolio B, Arlt W, et al. Diagnosis and treatment of primary adren
20insufficiency: an Endocrine Society Clinical Practice Guideline. J Clin Endocrinolal Metab. 2016;101(2):364-389.
- Jia X, Kowalski RG, Sciubba DM, Geocadin RG. Critical care of traumatic spinal cord injury. J Intensive Care Med. 2013;28(1):12-23.
465
-
Ziemssen T, Reichmann H. Treatment of dysautonomia in extrapyramidal disorders. Ther Adv Neurol Disord. 2010;3(1):53-67.
-
Aho JM, Thiels CA, El Khatib MM, et al. Needle thoracostomy: clinical effectiveness is improved using a longer angiocatheter. J Trauma Acute Care Surg. 2016;80(2):272-277.
-
Marriott HJL. Bedside Cardiac Diagnosis. Philadelphia, PA: Lippincott Company; 1993.
466
CHAPTER 23
467
Peripheral Edema
468 Case: A 47-year-old woman with palmar erythema A 47-year-old woman presents to the emergency department with progressive swelling of he legs over the past few months. Over the
same period of time shethas also experienced an increase in abdominal girth and a weight gain of 30 pounds. She reports drinking 2 bottles (1.5 L) o wine daily for the past 20 years.
8 Bloodfpressure is 98/57 mm Hg. Jugular venous pressurel(JVP) is 2 cm H O. Fingernails on both hands demonstrate white-co ored opacification involving the majority of the nail bed except the distal superficial veins radiating from the umbilicus. The abdomen is marked edema of the lower extremities. An indentation appears in the upper extremities. Photographs of the patient are shown in Figure 23- end where a strip of normal pink tissue remains. There are dilated symmetrically distended with dullness over the flanks. There is skin after pressure is applied over the tibia. There is no edema of the 1.
FIGURE 23-1
What are the most likely cause(s) of peripheral edema in this patient?
What is edema?
What is pitting edema? What is dependent edema? What is generalized edema? How is total body water distributed?
What factors regulate fluid balance between the intravascular and interstitial spaces?
Edema describes the presence of excess fluid within the interstitial space of the body. Peripheral edema involves the visible tissues (eg, extremities, sacral area, face, tongue, scrotum). Examples of nonperipheral edema include cerebral edema, pulmonary edema, pleural effusion, and ascites. Peripheral and nonperipheral edema may occur together, depending on the underlying condition. Pitting edema is characterized by the presence of a temporary indentation in the skin after firm pressure is applied to it. The severity of pitting edema can generally be graded on a scale from 1+(mild) to 4+(severe). A more precise way to grade pitting edema is to measure the depth of indentation over a bony prominence in millimeters. Dependent edema accumulates in the lower regions of the peripheral tissues due to gravity, such as the distal lower extremities in patients who are ambulatory, and the sacral area in patients who are bedbound. Dependent edema can be present in a localized (eg, unilateral upper extremity) or generalized distribution. Generalized edema (ie, anasarca) accumulates throughout the peripheral tissues, including gravity-dependent and gravity-independent regions. There is a dependent component to generalized edema, with more fluid accumulating in the gravity-dependent regions of the body. Generalized edema is frequently associated with pleural effusions and ascites. Two-thirds of total body water is distributed within the intracellular fluid compartment, and one-third is within the extracellular fluid compartment. The extracellular compartment can be further divided into the intravascular (one- fourth) and interstitial (three-fourths) spaces (see Figure 22-3).1 The movement of fluid between the intravascular and interstitial spaces is regulated by the interplay between hydrostatic pressures, oncotic pressures, capillary permeability, and the lymphatic system (Figure 23-2). The lymphatic system is responsible for returning net fluid efflux and filtered proteins from the interstitial space to the intravascular space. Lymphatic insufficiency causes proteins to accumulate within the interstitial space, increasing its oncotic pressure.1,2
469
FIGURE 23-2 The differential between hydrostatic and oncotic pressures at the arterial end of the capillary bed favors the
What are the 4 general mechanisms of edema formation?
movement of fluid into the interstitial space. The differential at the venous end of the capillary bed favors the reabsorption of most fluid back into the intravascular space. The lymphatic system reabsorbs fluid and proteins from the interstitial space. (Adapted from Rubin R, Strayer DS. Rubin’s Pathology: Clinicopathologic Foundations of Medicine. 5th ed. Philadelphia, PA: Lippincott Williams & Wilkins; 2008.)
Fluid accumulation within the interstitial space (ie, edema) can occur as a result of Increased capillary hydrostatic pressure, Decreased capillary oncotic pressure, Increased interstitial oncotic pressure, or Increased capillary permeability.
470
Peripheral Edema Related to Increased Capillary Hydrostatic Pressure
How does capillary hydrostatic pressure cause peripheral edema?
An increase in capillary hydrostatic pressure, which opposes capillary oncotic pressure and interstitial hydrostatic pressure, will lead to net efflux of fluid from the capillaries into the interstitial space. Edema forms when the maximal drainage capacity of the lymphatic system is exceeded (Figure 23-3A).
FIGURE 23-3 Mechanisms of edema formation. A, Increased capillary hydrostatic pressure. B,
Does systemic arterial hypertension cause peripheral edema? What are the general characteristics of peripheral edema caused by Increased capillary hydrostatic pressure?
Decreased capillary oncotic pressure. C, Increased interstitial oncotic pressure. D, Increased capillary permeability. (Adapted from Rubin R, Strayer DS.iRubin’s Pathology: Clinicopathologic Foundations of Medicine. 5th ed. Philadelph a, PA: Lippincott Williams & Wilkins; 2008.)
Systemic arterial hypertension does not cause peripheral edema because local autoregulation involving precapillary sphincters prevents arterial pressures from being directly transmitted to the capillary bed.1 Increased capillary hydrostatic pressure generally results in localized, dependent, pitting peripheral edema. It can be unilateral or bilateral depending on the underlying cause.
What are the causes of Peripheral Edema Related to Increased capillary hydrostatic pressure?
A 56-year-old man with a history of obstructive sleep apnea presents with elevated JVP, ascites, and peripheral edema. In addition to peripheral edema, other physical findings might include asterixis and a pericardial friction rub. Look for spider angiomas on the chest. A 55-year-old cancer patient develops acute onset unilateral lower extremity pain, erythema, and edema. Advanced disease is associated with ulcerations around
Right-sided heart failure.
Renal failure.
Cirrhosis.
Deep vein thrombosis (DVT).
Chronic venous insufficiency.
471
Affects women of Pregnancy. reproductive age. Iatrogenic Medication. peripheral edema. Pancoast tumor. Superior vena cava (SVC) syndrome. Elevated JVP, Constrictive pericarditis. Kussmaul’s sign, and an extra heart sound in early diastole best heard with the diaphragm of the stethoscope.
What are the characteristics of peripheral edema caused by heart failure?
What are the principles of managing hypervolemia in patients with renal failure? What are the characteristics of peripheral edema caused by cirrhosis? What are the characteristics of peripheral edema caused by deep vein thrombosis?
What is chronic venous insufficiency?
Heart failure causes peripheral edema as a result of Increased capillary hydrostatic pressure. In addition, ineffective arterial volume leads to renal retention of sodium and water, which contributes further to edema formation, usually involving the lower extremities. The edema is typically bilateral, symmetric, dependent, pitting, gradual in onset, and can be associated with ascites. Pulmonary edema or pleural effusions may be present in patients with left-sided heart failure. Diuretics, and dietary restriction of sodium and fluid are the mainstays of treatment. In some patients, bowel wall edema can limit diuretic medication absorption, necessitating use of the parenteral route.1
The characteristics of peripheral edema caused by renal failure parallel those of right-sided heart failure. Dietary restriction and fluid removal via hemodialysis or peritoneal dialysis are the cornerstones of treating hypervolemia and maintaining fluid balance in these patients. For those with severe manifestations of hypervolemia (eg, pulmonary edema), plasma ultrafiltration can be used to remove several liters of fluid per day.
Most fluid retention in patients with cirrhosis manifests as ascites, although peripheral edema tends to become prominent as the disease progresses. The edema is typically bilateral, symmetric, dependent, pitting, gradual in onset, and usually involves the lower extremities.1
The peripheral edema of DVT is typically unilateral, dependent, pitting, acute in onset, and often associated with pain and erythema. Compression Doppler ultrasonography is the diagnostic modality of choice. Anticoagulation is used to prevent complications in patients with proximal DVT (at or proximal to the level of the popliteal vein) and in some patients with distal DVT. In select patients with severe disease, thrombolytic therapy (eg, catheter-directed thrombolysis) can be considered. In patients with bilateral lower extremity edema, thrombosis of the inferior vena cava should be considered.3 Chronic venous insufficiency occurs when dysfunction of the venous system of the lower extremities leads to venous hypertension. This process most commonly develops as a result of venous valvular incompetence related to previous clinical or occult DVTs. Clinical sequelae include pain, edema, and skin changes such as telangiectasias, varicosities, hemosiderosis-related hyperpigmentation, lipodermatosclerosis (a fibrotic process involving the dermis and subcutaneous fat), and ulcerations (typically near the medial malleoli) (Figure 23-4). The edema is dependent and gradual in onset, and usually asymmetric although it can be unilateral or bilateral. Early in the course of the disease, it is soft and pitting but may become more resistant to palpation as it progresses. Compression stockings are first-line treatment.1,4
FIGURE 23-4 Hemosiderosis and an early ulceration near the medial malleolus (classic location) in a patient with chronic
How common is peripheral edema during pregnancy?
Which medications are most commonly
venous insufficiency. (From Goodheart HP. Goodheart’s Same-Site Differential Diagnosis: A RapidiMethod of Diagnosing and Treating Common Skin Disorders. Philadelphia, PA: Lippincott Will ams & Wilkins; 2011.)
Most pregnancies are complicated by peripheral edema, which occurs in the lower extremities in one-half of cases. Mechanisms of edema in pregnancy include Increased plasma volume and sodium retention, Decreased plasma protein concentration, and Increased capillary hydrostatic pressure from mechanical compression of the internal vena cava and iliac veins.1 Medications most often associated with peripheral edema include dihydropyridine calcium channel blockers (eg, amlodipine), direct vasodilators (eg, hydralazine), and nonsteroidal anti-inflammatory drugs (eg, ibuprofen).1,5
472
peripheral edema? Where does peripheral edema occur in patients with superior vena cava syndrome? What physical finding is useful for differentiating constrictive pericarditis from cirrhosis in patients presenting with ascites and peripheral edema?
Peripheral edema caused by SVC syndrome involves the face, neck, and upper extremities. Arm involvement tends to be bilateral, symmetric, dependent, and pitting.6
The clinical presentation of constrictive pericarditis can be similar to that of cirrhosis, including hepatic congestion, ascites, and peripheral edema. Elevated JVP is indicative of a cardiac condition, and in patients with constriction, the JVP typically increases with inspiration, a finding known as Kussmaul’s sign. For a video of Kussmaul’s sign, see the associated reference.7
473
Peripheral Edema Related to Decreased Capillary Oncotic Pressure
How does capillary oncotic pressure cause peripheral edema? What generates capillary oncotic pressure? What are the general characteristics of peripheral edema caused by Decreased capillary oncotic pressure?
Capillary oncotic pressure opposes capillary hydrostatic pressure and interstitial oncotic pressure to maintain fluid within the capillaries and promote reabsorption of interstitial fluid at the venous end of the capillary bed. A decrease in capillary oncotic pressure will lead to net efflux of fluid from the capillaries to the interstitial space (see Figure 23-3B). Plasma proteins, primarily albumin, generate capillary oncotic pressure. Plasma albumin concentration <2 g/dL is associated with peripheral edema.1,8 Decreased capillary oncotic pressure usually results in generalized, dependent, pitting peripheral edema. It is frequently associated with ascites and pleural effusions, but the presence of pulmonary edema suggests an alternative or additional disorder.9
What are the causes of Peripheral Edema Related to Decreased capillary oncotic pressure?
These conditions result in Decreased protein synthesis. These conditions result in abnormal protein loss from the body.
How much albumin is synthesized per day?
Liver disease and malnutrition.
Nephrotic syndrome and protein-losing enteropathy.
The liver is the only site of albumin synthesis. A healthy liver generates approximately 15 g of albumin daily. It can increase production by 2- to 3-fold when necessary. Decreased serum albumin concentration in patients with liver disease correlates with poorer prognosis. The fingernails can provide a clue to the presence of liver disease (Figure 23-5).10,11
FIGURE 23-5 Terry’s nails describes white-colored opacification of most of the nail bed, sparing a narrow 1 to 2 mm band of normal pink to brown tissue at the distal end. It is a sign of systemic disease (eg, cirrhosis).
What are the physical findings of protein-calorie malnutrition?
What are the characteristic urinary findings of
Protein-calorie malnutrition results in atrophy of muscle and fat, and peripheral edema. Clues to the presence of muscle atrophy include hollowed temples and rib protrusion in the pectoralis muscle area. Atrophy of the temporalis muscle is particularly informative because other muscles in the body (eg, quadriceps) may atrophy as a result of disuse, independent of nutritional status. Signs of subcutaneous fat atrophy include hollowed appearance and dark circles in the orbital region, marked intercostal depressions, and prominent iliac crests. Kwashiorkor is a pediatric condition caused by severe protein-calorie malnutrition and is characterized by dermatitis, protuberant abdomen, thinning hair, and peripheral edema (see Figure 13-4).12,13 Nephrotic syndrome is characterized by proteinuria of at least 3.5 g/d. A minority of patients may also experience microscopic hematuria, but the urine sediment is typically bland. The peripheral edema of nephrotic syndrome is usually generalized, dependent, and pitting.14
474
nephrotic syndrome? What is the initial test of choice to evaluate for protein-losing enteropathy?
The initial diagnostic test of choice for protein-losing enteropathy is measurement of fecal clearance of α-1 antitrypsin, which is elevated in affected patients. Characteristic clinical manifestations include peripheral edema, ascites, pleural effusions, and pericardial effusion.15
475
Peripheral Edema Related to Increased Interstitial Oncotic Pressure
How does Increased interstitial oncotic pressure cause peripheral edema? What generates interstitial oncotic pressure?
What are the general characteristics of peripheral edema caused by Increased interstitial oncotic pressure?
Interstitial oncotic pressure opposes capillary oncotic pressure and interstitial hydrostatic pressure to draw fluid into the interstitial space and prevent its reabsorption at the venous end of the capillary bed. An increase in interstitial oncotic pressure will lead to net efflux of fluid from the capillaries to the interstitial space (see Figure 23-3C). Interstitial oncotic pressure is primarily generated by glycosaminoglycans (ie, mucopolysaccharides) and filtered proteins (eg, albumin), the concentrations of which are influenced by capillary wall protein permeability and the rate of lymphatic clearance.1 Increased interstitial oncotic pressure generally results in localized, dependent, nonpitting peripheral edema. It can be unilateral or bilateral, depending on the underlying cause.
What are the causes of Peripheral Edema Related to Increased interstitial oncotic pressure?
The most common causes of this condition are malignancy (in the industrialized world) and infection (worldwide). A 45-year-old woman presents with weight gain, constipation, dry hair, bradycardia, and bilateral nonpitting lower extremity edema.
Lymphedema.1
Myxedema.
Why does Lymphatic obstruction leads to retention of protein-rich fluid in the interstitium, generating an increase in interstitial lymphatic oncotic pressure. The most common causes include malignant obstruction (eg, lymphoma), iatrogenesis (surgery obstruction involving lymphatics, radiation treatment), and infection (eg, filariasis). The peripheral edema occurs on the ipsilateral cause an side and is usually dependent. Pitting occurs early in the course of the disease. Over time the skin becomes thickened and increase in darkened, and may develop warty projections (ie, lymphostatic verrucosis). Treatment options, including diuretics, are interstitial usually unsuccessful. Lipedema refers to abnormal deposition of fatty substances in the subcutaneous tissues that results in soft oncotic tissue swelling, which is frequently mistaken for lymphedema. It predominates in women and almost always involves the lower pressure? extremities (but usually spares the feet).1,5,16 What are the Myxedema can develop in the setting of hypothyroidism or hyperthyroidism but is more common in the former. characteristics Hypothyroidism causes Increased capillary permeability and Decreased lymphatic clearance, leading to the accumulation of myxedema? of glycosaminoglycans and albumin in the interstitial space. This generates nonpitting peripheral edema that most often involves the lower extremities but may become generalized or involve nondependent areas, such as the eyelids, face, or dorsum of the hands. A particular type of myxedema occurs in patients with Graves’ disease, called Graves’ dermopathy. It results from an inflammatory-mediated accumulation of glycosaminoglycans in the interstitial space. It develops over the pretibial region in the vast majority of cases (ie, pretibial myxedema) and is characterized by bilateral, asymmetric, nondependent, nonpitting, painless, often discolored (yellow-brown to erythematous) nodules and plaques. Over time the lesions may coalesce to produce symmetric involvement of the pretibial regions, which become thickened and hardened, taking on a peau d’orange (orange peel) appearance and texture (Figure 23-6).1,17–19
476
FIGURE 23-6 Graves’ dermopathy (ie, pretibial myxedema). There are erythematous plaques in the pretibial region. Note the peau d’orange (orange peel) appearance in some areas. (From Goodheart HP. Goodheart’s Same-Site Differential Diagnosis: A RapidiMethod of Diagnosing and Treating Common Skin Disorders. Philadelphia, PA: Lippincott Will ams & Wilkins; 2011.)
477
Peripheral Edema Related to Increased Capillary Permeability
How does Increased capillary permeability cause peripheral edema? What are the general characteristics of peripheral edema caused by Increased capillary permeability?
The capillary membrane maintains fluid within the intravascular space and prevents equilibration of large proteins between the intravascular and interstitial spaces, which would diminish the oncotic pressure gradient that inhibits the movement of fluid out of the capillaries. An increase in permeability leads to net efflux of fluid and proteins from the capillaries to the interstitial space (see Figure 23-3D). Increased capillary permeability can be associated with a variety of phenotypes, including pitting or nonpitting peripheral edema that is localized or generalized depending on the underlying etiology.
What are the causes of Peripheral Edema Related to Increased capillary permeability?
Local cytokine release. A 29-year-old pregnant woman in the 35th week of gestation develops hypertension, proteinuria, and peripheral edema. A 46-year-old man with hypertension develops swelling of the tongue, lips, and hands after starting a new antihypertensive medication. A 28-year-old woman with unexplained episodes of edema involving the hands, legs, and abdomen despite a thorough diagnostic investigation. A rare disorder characterized by episodes of hypotension, hemoconcentration, and peripheral edema.
What causes of local inflammation can result in edema? What are the characteristics of peripheral edema caused by preeclampsia? What are the characteristics of angioedema?
What are the characteristics of idiopathic edema?
What is systemic capillary leak syndrome?
Local inflammation. Preeclampsia.
Angioedema caused by use of an angiotensin-converting enzyme inhibitor.
Idiopathic edema.
Systemic capillary leak syndrome.
Peripheral edema can occur as a result of trauma, stinging insects (eg, bees, wasps), infection (eg, cellulitis), or burn injury. It is typically localized and nonpitting. History is key in making the diagnosis. In trauma patients, limb pain, paresthesias, tense edema, and rising serum creatine kinase levels suggests acute compartment syndrome, a surgical emergency. The combination of hypoalbuminemia and endothelial dysfunction with Increased capillary permeability causes peripheral edema in patients with preeclampsia. It tends to involve the lower extremities and may be accompanied by pulmonary and cerebral edema. Management depends on severity and gestational age. Clinical manifestations typically resolve within hours of delivery.20 Angioedema results in localized, nondependent, and nonpitting peripheral edema, most often involving the face (eg, lips, tongue, periorbital region), hands, feet, or genitalia. Increased capillary permeability is usually mediated by histamine or bradykinin. Common causes include allergies (eg, food, venom), medications (eg, angiotensin-converting enzyme inhibitors, nonsteroidal anti-inflammatory drugs), and environmental triggers (eg, cold urticaria, exercise-induced anaphylaxis).21 Idiopathic edema most commonly affects menstruating women in the third and fourth decades of life. It is periodic in nature but not clearly associated with the menstrual cycle. Edema develops in the upright position as the day progresses, most often affecting the lower extremities, face, and hands. Patients frequently misuse diuretics or laxatives in an effort to treat the condition. Idiopathic edema is different than premenstrual edema, which occurs in most women in the days before the onset of menses and resolves with menses.1,5,22 Systemic capillary leak syndrome is a rare and life-threatening disorder that typically affects middle-aged adults. It is characterized by episodes of rapid- onset generalized edema and hypovolemic shock associated
478
with manifestations of hemoconcentration (eg, elevated hematocrit) and hypoalbuminemia without albuminuria. The edema may be pitting or nonpitting. The pathogenesis of Increased capillary permeability in this condition is not known. A monoclonal gammopathy is present in most patients. 23
479 Case Summary A 47-year-old woman with chronic heavy alcohol use presents with
increasing abdominal girth, weight gain, and lower extremity edema. What are the most likely cause(s) of peripheral edema in this patient? Cirrhosis with or without hypoalbuminemia.
480
Bonus Questions
What are the s The peripheral edema in this caselis localized to the lower extremities, dependent, and pitting, suggesting that the underlying cause is related to Increased capillary peripheral of the characteristic hydrostatic pressure. The bilatera location makes DVT unlikely, further narrowing the differential diagnosis. edema described in this case? What is the Terry s nails describes white-colored opacification of most of the nail bed, spar ng a narrow 1 to 2 mm band of normal pink to brown tissue at the distal end (see Figu significance of 23-4).’The opacification results in disappearance of the lunula. Terry’s nails areia sign of systemic disease, such as cirrhosis, chronic heart failure, and chronic kidney re the fingernail disease. 24 findings in this case? What is the of Right-sided heart failure and constrictive pericarditis can mimic the presentation of cirrhosis. The normal JVP in this case indicates that cardiac and renal causes of the jugular venous significance Increased capillary hydrostatic pressure are unlikely.
pressure in this case? What are the Figure 23-1 demonstrates jaundice, scleral icterus, palmar erythema, and spider angiomas on the chest, all of which are associated with advanced liver disease. Palmar relevant erythema describes erythema of the thenar and hypothenar eminences of the hands with central pallor. Spider angiomas are cutaneous vascular lesions that most often physical appear on the upper trunk. There is a central arteriole from which small branching vessels radiate. The lesion will blanch with pressure and fill from the central arteriole
findings in? outward. Other stigmata of advancediliver disease includeiascites, caput medusae (tortuous superficial veins that radiate from the umbilicus), and gynecomastia in men. most likelyers disease suggest progression tolcirrhosis (Laennec’s cirrhosis). The risk of developing cirrhosis becomes significant with chronic’consumption of >30 g of alcohol per day Figure 23-1 The hollowed temples and orbital reg ons suggest malnutr tion; hypoalbuminemia from the combination of liver disease and malnutrition is likely in this case. 25 What is the The history of chronic heavy a cohol use by the patient in this case is suggestive of alcoholic liver disease; the presence of Terry s nails and stigmata of advanced liver case? cause of liv (a 750-mL bottle of wine contains 75 g of alcohol); risk rises proportionally with intake greater than 30 g. Even in patients with a history compatible with Laennec’s disease in thi cirrhosis, other common etiologies of cirrhosis (eg, chronic hepatitis B or C virus infection) should be ruled out. 26,27 How is the Liver biopsy is the gold standard for diagnosing cirrhosis but is seldom required. The diagnos s is most often made clinically based on characteristic history, diagnosis of examination, biochemical laboratory data (eg, hypoalbuminemia, elevated prothrombin time),iand imaging data (eg, evidence of liver nodularity, signs of portal cirrhosis hypertension). 28 made? What are the Conservative nonpharmacologic management of peripheral edema in cirrhotics includes dietary sodium restriction, leg elevation, and compression stockings. Diuretic options for the spironolactone:furosemide dosing ratio of 50:20 mg is used to maintain serum potassium (at these doses, the potassium-wasting effect of furosemide is counterbalanced edema in this bioavailability of oral diuretics is usually preserved in cirrhotics.1,29,30 treatment therapy is often necessary. The combination of spironolactone and furosemide is particularly effective, usually starting at 100 mg and 40 mg, respectively. The peripheral by the potassium-sparing effect of spironolactone). Unlike some patients with heart failure who experience impaired absorption related to bowel wall edema, the case?
481 Key Points
Peripheral edema describes the presence of excess fluid in the interstitial space. The movement of fluid between the intravascular and interstitial
spaces is regulated byithe interplay between hydrostatic pressures, Peripheral edema can be caused by Increased capillary hydrostatic interstitial oncotic pressure, or Increased capillary permeability. within the interstitial space, increasing its oncotic pressure. localized, dependent, pitting peripheral edema. It can be unilateral Decreased capillary oncotic pressure usually results in generalized, oncotic pressures, cap llary permeability, and the lymphatic system. pressure, Decreased capillary oncotic pressure, Increased Lymphatic insufficiency leads to the accumulation of proteins Increased capillary hydrostatic pressure generally results in or bilateral, depending on the underlying cause. dependent, pitting peripheral edema. It is frequently associated with ascites and pleural effusions.
Increased interstitial oncotic pressure generally results in localized, dependent, nonpitting peripheral edema. It can be unilateral or bilateral, depending on the underlying cause.
Increased capillary permeability can be associated with a variety of a localized or generalized distribution, depending on the phenotypes, including pitting or nonpitting peripheral edema with underlying etiology.
482
References 1. Cho S, Atwood JE. Peripheral edema. Am J Med. 2002;113(7):580-586. 2. Warren AG, Brorson H, Borud LJ, Slavin SA. Lymphedema: a comprehensive review. Ann Plast Surg. 2007;59(4):464-472.
-
Kesieme E, Kesieme C, Jebbin N, Irekpita E, Dongo A. Deep vein thrombosis: a clinical review. J Blood Med. 2011;2:59-69.
-
Eberhardt RT, Raffetto JD. Chronic venous insufficiency. Circulation. 2014;130(4):333- 346.
-
Ely JW, Osheroff JA, Chambliss ML, Ebell MH. Approach to leg edema of unclear etiology. J Am Board Fam Med. 2006;19(2):148-160.
-
Cheng S. Superior vena cava syndrome: a contemporary review of a historic disease. Cardiol Rev. 2009;17(1):16-23.
-
Mansoor AM, Karlapudi SP. Images in clinical medicine. Kussmaul’s sign. N Engl J Med. 2015;372(2):e3.
-
Weisberg HF. Osmotic pressure of the serum proteins. Ann Clin Lab Sci. 1978;8(2):155- 164. Zarins CK, Rice CL, Peters RM, Virg lio RW. Lymph and pulmonary response to
-
isobaric reduction in plasma oncoticipressure in baboons. Circ Res. 1978;43(6):925-930. 10. Limdi JK, Hyde GM. Evaluation of abnormal liver function tests. Postgrad Med J. 2003;79(932):307-312.
-
Nicholson JP, Wolmarans MR, Park GR. The role of albumin in critical illness. Br J Anaesth. 2000;85(4):599-610.
-
Bharadwaj S, Ginoya S, Tandon P, et al. Malnutrition: laboratory markers vs nutritional assessment. Gastroenterol Rep (Oxf). 2016;4(4):272-280. . Tierney EP, Sage RJ, Shwayder T. Kwashiorkor from a severe dietary restriction in an
138-month infant in suburban Detroit, Michigan: case report and review of the literature. Int J Dermatol. 2010;49(5):500-506.
-
Hull RP, Goldsmith DJ. Nephrotic syndrome in adults. BMJ. 2008;336(7654):1185-1189. 15. Umar.SB, DiBaise JK. Protein-losing enteropathy: case illustrations and clinical review Am J Gastroenterol. 2010;105(1):43-49; quiz 50. . Forner-Cordero I, Szolnoky G, Forner-Cordero A, Kemeny L. Lipedema: an over 16of its clinical manifestations, diagnosis and treatment of the disproportional fatty view deposition syndrome – systematic review. Clin Obes. 2012;2(3-4):86-95.
-
Doshi DN,tBlyumin ML, Kimball AB. Cutaneous manifestations of thyroid disease. 18of edema formation in myxedema–Increased protein extravasation and relatively slow Clin Derma ol. 2008;26(3):283-287. . Parving HH, Hansen JM, Nielsen SL, Rossing N, Munck O, Lassen NA. Mechanisms lymphatic drainage. N Engl J Med. 1979;301(9):460-465.
-
Wolff K, Johnson RA, Saavedra AP. Fitzpatrick’s Color Atlas andlSynopsis of Clinical 013. 20pathophysiology, diagnosis, and management. Vasc Health Risk Manag. 2011;7:467- Dermatology. 7th ed. United States of America: The McGraw-Hil Companies, Inc.; 2 . Uzan J, Carbonnel M, Piconne O, Asmar R, Ayoubi JM. Pre-eclampsia: 474.
-
Kaplan AP. Angioedema. World Allergy Organ J. 2008;1(6):103-113.
- Coleman M, Horwith M, Brown JL. Idiopathic edema. Studies demonstrating protein-
483
leaking angiopathy. Am J Med. 1970;49(1):106-113. 23(Clarkson’s disease): the Mayo clinic experience. Mayo Clin Proc. 2010;85(10):905-912. 24. Witkowska AB, Jasterzbski TJ, Schwartz RA. Terry’s nails: a sign of systemic disease. . Kapoor P, Greipp PT, Schaefer EW, et al. Idiopathic systemic capillary leak syndrome Indian J Dermatol. 2017;62(3):309-311.
-
Karnath B. Stigmata of chronic liver disease. Hosp Physician. 2003;39(14):14-16, 28. 26. Bellentani S, Saccoccio G, Costa G, et al. Drinking habits as cofactors of(risk for 50. 27. Lucey MR, Mathurin P, Morgan TR. Alcoholic hepatitis. N Engl J Med. alcohol induced liver damage. The Dionysos Study Group. Gut. 1997;41 6):845-8 2009;360(26):2758-2769.
-
Tsochatzis EA, Bosch J, Burroughs AK. Liver cirrhosis. Lancet. 2014;383(9930):1749- 1761. . Fogel MR, Sawhney VK, Neal EA, Miller RG, Knauer CM, Gregory PB. Diuresis in
29ascitic patient: a randomized controlled trial of three regimens. J Clin Gastroenterol.the 1981;3(suppl 1):73-80.
- Sawhney VK, Gregory PB, Swezey.SE, Blaschke TF. Furosemide disposition in cirrhotic patients. Gastroenterology 1981;81(6):1012-1016.
484
CHAPTER 24
485
Syncope
486 Case: A 64-year-old man with an enlarged cardiac silhouette A 64-year-old man with a history of metastatic renal cell carcinoma headedness. Symptoms have progressed over the course of a week presents to the emergency department with dyspnea and light- and culminated in an episode of lost consciousness on the day of presentation. He denies any chest pain or palpitations. His wife
witnessed the episode and did not observe any seizure-like activity. to be confused on arousal. The patient was diagnosed with metastatic cytoreductive nephrectomy and a tyrosine kinase inhibitor. Despite new bone lesions. He was unconscious for approximately 30 seconds and did not appear renal cell carcinoma 10 months ago and has been treated with therapy, recent imaging revealed enlarging lymphadenopathy and Heart rate is 104 beats per minute, and blood pressure is 98/61 mm Hg. Jugular venous pressure is elevated to 16 cm H O with 2 an attenuated Y descent. Heart sounds are distant. Lungs are clear. i Chest radiograph is shown in Figure 24-1A (for comparison, Electrocardiogram shows low voltage and sinus tachycardia. maging from 2 weeks prior is shown in Figure 24-1B).
FIGURE 24-1
What is the most likely cause of syncope in this patient?
What is syncope?
What is the hemodynamic mechanism of most causes of cerebral hypoxia? How common is syncope?
What is the prognosis of syncope? The causes of syncope can be separated into which general categories?
Syncope is the transient loss of consciousness resulting from cerebral hypoxia, with spontaneous revival. Conditions that cause loss of consciousness through other mechanisms will be included within the framework of this chapter.1 Cerebral hypoxia most often occurs as a result of Decreased cerebral blood flow.
Syncope occurs in one-third of individuals in the general population at some point in life. First-time episodes of syncope follow a bimodal distribution, with peaks at ages 20 and 80 years. In the industrialized world, syncope accounts for 1% of emergency department visits. Around one-third of those patients are subsequently admitted to the hospital.1 The prognosis of syncope varies widely based on the underlying cause, from benign (eg, vasovagal syncope) to life-threatening (eg, cardiac tamponade). The causes of syncope can be separated into the following categories: cardiovascular, neurocardiogenic, neurologic, and other.
487
488
Cardiovascular Causes of Syncope
What is the mechanism of most cardiovascular causes of syncope? What are the main determinants of cardiac output?
Cardiovascular syncope generally occurs as a result of Decreased cardiac output (CO). Cardiac output is equal to the forward stroke volume (SV) of the left ventricle per beat multiplied by heart rate (HR).2CO =SV ×HR
What are the cardiovascular causes of syncope?
Low central venous pressure, Hypovolemia. low cardiac output, and elevated systemic vascular resistance. An episode of syncope is Dysrhythmia. preceded by palpitations. Pulsus alternans and cold Heart failure. extremities. Levine’s sign. Acute myocardial infarction (MI). A 60 year-old woman from Pulmonary embolism (PE). Germany presents with leg swelling and syncope the day after arriving in Portland, Oregon, to visit her son. These conditions are Valvular lesions and left ventricular outflow tract obstruction (eg, hypertrophic obstructive associated with murmurs on cardiomyopathy [HOCM]). cardiac auscultation. Tearing chest pain associated Aortic dissection. with blood pressure of 185/110 mm Hg in one arm and 90/60 mm Hg in the other arm. A strong pulsation is palpated Pulmonary hypertension. over the left second intercostal space of the chest. Elevated central venous Cardiac tamponade. pressure, low cardiac output, and muffled heart sounds. A young woman with Subclavian steal syndrome. Takayasu arteritis complains of recurrent episodes of syncope when she does upper arm exercises.
What are the causes of hypovolemia? What is the treatment for heart block associated with syncope? Which class of medications can be used to directly increase cardiac output in patients with cardiogenic shock?
What widely available treatment should be given immediately to patients who present with hypotension or syncope in the setting of a right ventricular infarction?
Hypovolemic hypotension can occur as a result of poor oral intake, Gastrointestinal fluid loss (eg, diarrhea, vomiting), renal loss (eg, diuretic use, primary adrenal insufficiency), hemorrhage, burns, or insensible loss (eg, excessive perspiration). In cases of symptomatic heart block (eg, third-degree AV block), urgent pharmacologic treatment (eg, atropine) or temporary cardiac pacing should be pursued. Unless a reversible cause can be addressed, placement of a permanent pacemaker is usually necessary.3,4 Inotropic agents (eg, dobutamine) can increase myocardial contractility and are useful for treating cardiogenic shock. Additional support with vasopressor agents (eg, norepinephrine) may also be helpful. Mechanical support with intra-aortic balloon counterpulsation (ie, intra-aortic balloon pump) may be beneficial in select patients with cardiogenic shock. It improves coronary and peripheral perfusion by increasing diastolic pressure via diastolic balloon inflation, and augments left ventricular performance by decreasing afterload via systolic balloon deflation.5 Patients with right ventricular infarction are particularly sensitive to inadequate preload. Intravenous crystalloid fluid should be given to patients with hemodynamic instability to increase preload and right ventricular contractility. If optimization of preload does not improve hemodynamics, inotropic support (eg, dobutamine) can be effective. Agents that decrease preload (eg, nitrates, diuretics) should be avoided. Early coronary reperfusion (eg, percutaneous coronary intervention) improves right ventricular performance and survival.6
489
acute pulmonary embolism hemodynamic instability? complicated by Which valvular lesion most frequently causes syncope?
fluids and vasopressors. In patients without contraindications, systemic thrombolytic therapy should be administered. Embolectomy (surgical or catheter-based) may be considered when there is contraindication to, or failure of, thrombolytics. 7 Syncope is one of the classic clinical manifestations of severe aortic stenosis. The other common manifestations include chest pain and dyspnea. Without treatment, the mean survival in patients with syncope caused by severe aortic stenosis is 3 years.8
What classes of medications can precipitate syncope in patients with hypertrophic obstructive cardiomyopathy? What are the risk factors for aortic dissection? Which echocardiographic measurement is used to assess pulmonary arterial pressure? What blood pressure finding is associated with cardiac tamponade?
What is subclavian steal syndrome?
Pharmacologic agents that may increase outflow obstruction in patients with HOCM include diuretics (via volume depletion), vasodilators (via Decreased preload), and inotropic agents (via Decreased left ventricular end-diastolic volume as a result of Increased contractility).9
Risk factors for aortic dissection include male sex, age in the 60s and 70s, hypertension, prior cardiac surgery (especially aortic valve repair), bicuspid aortic valve, and Marfan syndrome.10 Echocardiography can estimate right ventricular systolic pressure, which can be used as a surrogate for pulmonary arterial pressure. Calculating the right ventricular systolic pressure requires an estimate of right atrial pressure and the presence of tricuspid regurgitation. Based on the estimated right atrial pressure (P) and the measured velocity (V) of the regurgitant jet, the simplified Bernoulli equation (ΔP =4V2) can be used to calculate the difference between right ventricular systolic pressure and estimated right atrial pressure.11 Cardiac tamponade is associated with pulsus paradoxus, which is a drop in systolic blood pressure ≥10 mm Hg during inspiration. To measure the pulsus, the cuff should be inflated above the highest systolic pressure. The patient should breath naturally. The cuff is then slowly deflated until the first Korotkoff sounds are heard (pressure A). These sounds will be appreciable during expiration only, when systolic arterial pressure is highest and able to overcome the cuff pressure. The cuff should be deflated further until the Korotkoff sounds are appreciable during both expiration and inspiration (pressure B). The difference between pressures A and B is the value of the pulsus.12 Subclavian steal syndrome describes retrograde blood flow in the vertebral artery as a result of a hemodynamically significant proximal narrowing of the ipsilateral subclavian artery. When the pressure distal to the site of stenosis falls below the pressure of the contralateral vertebral and internal carotid arteries there will be flow reversal down the ipsilateral vertebral artery, “stealing” blood from the cerebral circulation (Figure 24-2). When perfusion to the posterior brain is significantly compromised, symptoms such as light-headedness and syncope may appear. Upper arm activity can precipitate symptoms by increasing blood flow to the arm. Atherosclerosis is the most common cause of subclavian stenosis. Other causes should be considered in younger patients with subclavian steal syndrome, including large vessel vasculitis, thoracic outlet syndrome, and postsurgical stenosis (eg, after aortic coarctation repair).13
FIGURE 24-2 Subclavian steal syndrome. Proximal occlusion of the left subclavian artery causes retrograde blood flow through the left vertebral artery, “stealing” blood from the basilar circulation and causing transient dizziness and syncope with arm exercise. (From Lawrence PF, Bell RM, Dayton MT. Essentials of General Surgery. 5th ed. Philadelphia, PA: Lippincott Williams & Wilkins; 2013.)
490
Neurocardiogenic Causes of Syncope
What is neurocardiogenic syncope?
Neurocardiogenic syncope occurs as a result of either a neurally mediated reflex that results in bradycardia and vasodilation, or impairment of the normal neurally mediated compensatory reflex that maintains blood pressure. Neurocardiogenic syncope is sometimes called “reflex” syncope.14
What are the neurocardiogenic causes of syncope?
A young boy passes out at the sight of a needle before receiving a vaccine. A previously healthy 46-year-old woman experiences recurrent episodes of syncope that only occur in the restroom. A 66-year-old man complains when his wife asks him to shave because sometimes it causes him to pass out. Labile blood pressure in the elderly. This form of autonomic dysfunction is most commonly seen in children and young adults. A consequence of spinal trauma.
How common is vasovagal syncope?
What are the scenarios in which situational syncope occurs? What is the mechanism of carotid hypersensitivity?
In patients with dysautonomia, what pattern of blood pressure and heart rate changes occur with tilt table testing? What patient demographic is most commonly affected by postural orthostatic tachycardia syndrome?
Vasovagal syncope.
Situational syncope.
Carotid hypersensitivity.
Dysautonomia. Postural orthostatic tachycardia syndrome (POTS).
Neurogenic shock.
Vasovagal syncope is the most common cause of syncope in the general population, accounting for around one-quarter of syncopal events. Clues to the presence of vasovagal syncope include onset during the teenage years, a family history of similar episodes, and association with emotional events.15,16 Situational syncope can occur with coughing, swallowing, laughing, micturition, or defecation. First-line therapy is avoidance of known triggers when possible.14
Syncope may occur when bradycardia and hypotension develop as a result of an exaggerated response to mechanical stimulation of the baroreceptors within the carotid sinus. It is rare in patients under 40 years of age; prevalence increases with age and cardiovascular comorbidities. In addition to shaving, it may also occur with rotation or turning of the head or use of tight collars or neckwear. Carotid sinus massage (for 5-10 seconds) can be diagnostic when it results in ≥50 mm Hg reduction in systolic blood pressure or a ventricular pause of ≥3 seconds.14 In patients with dysautonomia, tilt table testing results in a drop in blood pressure with little change in heart rate. This pattern distinguishes dysautonomia from orthostatic hypotension, a common cause of syncope, in which there is a compensatory increase in heart rate when blood pressure drops (except in patients receiving negative chronotropic therapy; eg, β-blockers).17
POTS overwhelmingly affects young women. The pathophysiology of POTS has not been fully elucidated. In addition to orthostatic symptoms, patients often experience other autonomic abnormalities (eg, hyperhidrosis), fatigue, migraine headache, and sleep abnormalities. Tilt table testing typically results in an exaggerated increase in heart rate. Pharmacologic agents that may be effective in treating symptoms of this condition include β- blockers, fludrocortisone, midodrine, and selective serotonin reuptake inhibitors. 18
491
dysrhythmia frequently develops in patients with neurogenic shock?
levels of the cervical cord are involved. Pharmacologic management (eg, atropine) or temporary pacing may be necessary in these patients to improve hemodynamics. 19
492
Neurologic Causes of Syncope
What are the neurologic causes of syncope?
Loss of consciousness associated with uncontrollable myoclonic jerking. This condition must be considered in any diabetic patient presenting with syncope. Loss of consciousness associated with focal neurologic symptoms or signs. A 78-year-old man loses consciousness when attempting to change a light bulb in his kitchen ceiling.
What historical features can help distinguish seizure from true syncope (related to Decreased cerebral blood flow)? What are the symptoms and signs of neuroglycopenia?
What additional symptoms and signs are present in patients with syncope related to transient ischemic attack or stroke? Why would the act of changing a light bulb precipitate syncope in patients with vertebrobasilar insufficiency?
Seizure.
Neuroglycopenia (hypoglycemia).
Transient ischemic attack (TIA) or stroke.
Vertebrobasilar insufficiency.
Compared with true syncope, features that favor seizure include a sense of déjà vu or jamais vu preceding an episode, evidence of tongue biting, witnessed head turning or posturing during an episode, and a postictal state. The absence of sweating, a history of light-headedness, or orthostatic position changes further support the diagnosis.20 Manifestations of neuroglycopenia range from behavioral changes, fatigue, and confusion to seizure and syncope. Hypoglycemia should be treated urgently with the ingestion of carbohydrates if possible, or the administration of parenteral glucagon or glucose. Reversal of hypoglycemia results in complete recovery in most cases. However, severe and prolonged hypoglycemia can result in brain death.21 Patients with syncope related to TIA or stroke tend to have concurrent neurologic manifestations associated with compromise of the posterior circulation, including vertigo (most common), ataxia, paresthesia, diplopia, nausea or vomiting, dysarthria, headache, dysphagia, and paresis. The prototypical patient is an older man with a history of hypertension and ischemic heart disease.22 In patients with vertebrobasilar insufficiency, turning the head upward creates external mechanical forces on the already-compromised vertebrobasilar arteries, which result in further decreases in cerebral blood flow. Syncope results when there is poor perfusion to the reticular activating system, located in the paramedian tegmentum of the upper brain stem.23
493
Other Causes of Syncope
What are the other causes of syncope?
A 76-year-old man experiences light-headedness and syncope after starting treatment for benign prostatic hyperplasia. In these conditions, oxygen delivery to the brain is compromised even in the setting of normal blood flow. Low carbon dioxide levels in arterial blood. Hyperresonance to percussion on one side of the chest. Think of this condition in patients with a mental health disorder in whom an etiology of recurrent syncope-like episodes cannot be determined.
α-Blocker medication.
Hypoxemia and anemia.
Hyperventilation.
Tension pneumothorax.
Psychogenic pseudosyncope.
What medications Diuretics and nitrates are the medications most strongly associated with syncope; others include sedatives, centrally most often cause acting α-adrenergic blockers, peripherally acting α-adrenergic blockers, vasodilators, and β-blockers.24,25 or precipitate syncope? What formula The significance of anemia and hypoxemia can be demonstrated in the formula that describes the delivery of oxygen describes the (DO ) to the tissues:DO =[1.34 ×Hb ×Sao +(0.003 ×Pao )] ×CO 2 2 2 2 effects of In the above formula, 1.34 is the oxygen-binding capacity of hemoglobin (mL O2/g of Hb), Hb is the hemoglobin concentration in blood (g/dL), Sao2 is the fraction of oxygenated hemoglobin in arterial blood, 0.003 is the solubility coefficient of O2 in blood hypoxemia and anemia on the delivery of (mL O2/100 mL blood/mm Hg Pao2), Pao2 is the partial pressure of oxygen in arterial blood (mm Hg), and CO is cardiac output oxygen to the (L/min).26 tissues? What is the Hyperventilation leads to hypocapnia, which causes cerebral vasoconstriction and an associated decrease in cerebral mechanism of blood flow. For every 1 mm Hg drop in Pa , there is a 2% decrease in cerebral blood flow.27 co 2 syncope related to hyperventilation? What is the Tension pneumothorax compresses the heart and other mediastinal structures, impeding venous return and mechanism of decreasing cardiac output (Figure 24-3). Emergency needle thoracostomy is the treatment of choice. syncope related to tension pneumothorax?
494
FIGURE 24-3 AP supine chest radiograph of a 35-year-old man involved in a motor vehicle accident shows a large left
What is psychogenic pseudosyncope?
pneumothorax, collapse of the left lung, depressed left hemidiaphragm, and rightward shift of the mediastinum. The findings suggest a tension pneumothorax, which requires immediate decompression. (From Collins J, Stern EJ. Chest Radiology: The Essentials. Philadelphia, PA: Wolters Kluwer Health; 2015.)
Psychogenic pseudosyncope is a type of conversion disorder characterized by the appearance of syncope in the absence of true loss of consciousness. The absence of seizure-like body movement differentiates it from psychogenic nonepileptic seizures. The prototypical patient is a young woman with increasingly frequent episodes over a period of several months. Patients often experience prodromal symptoms such as light-headedness, palpitations, chest pain, dyspnea, and tingling. During episodes, the eyes are often closed, in contrast to patients with true syncope. History is key to making the diagnosis. However, head-up tilt testing, electroencephalography, and transcranial Doppler ultrasonography can confirm the absence of cerebral hypoperfusion or other hemodynamic derangements characteristic of syncope (eg, hypotension, bradycardia) during an episode.28
495 Case Summary A 64-year-old man with a history of metastatic renal cell carcinoma presents with syncope and is found to have tachycardia, hypotension
elevated jugular venous pressure, muffled heart sounds, low-voltage, ECG, and an abnormal chest radiograph.
What is the most likely cause of syncope in this patient? Cardiac tamponade.
496
explanation for the Bonus Questions
What is the most likely pericardial effusion in this
electrocardiographic case? What are the features of cardiac tamponade? What relevant finding is radiograph in this case? cardiac silhouette in this a pericardial effusion than syncope in the setting of f present on the chest Why is the enlarged case more likely caused by cardiomegaly? What is the mechanism o cardiac tamponade?
What is the mechanism of pulsus paradoxus?
strategies for cardiacnt What are the treatme tamponade?
The pericardial effusion in this case is most likely malignant, related to metastatic renal cell carcinoma.
Sinusitachycardia is the most common electrocardiographic manifestation of cardiac tamponade. More specific findings include reduced voltage and electr cal alternans (alternating amplitude of the QRS complex). 12
The chest radiograph in this case (see Figure 24-1A) shows an enlarged cardiac silhouette, consistent with pericardial effusion. For the cardiac silhouette to increase, effusions must be large, typically >250 mL. Significantly smaller effusions can be detected by echocardiography. 12 subacute process that is more consistent with pericardialieffusion, as,cardiomegaly generally develops over a longer period of time. Furthermore, clear lungs When compared to the historical chest radiograph (see F gure 24-1B) the recent chest radiograph (see Figure 24-1A) in this case indicates an acute or would not be expected in patients with recent development of cardiomegaly and congestive heart failure.
In patients with cardiac tamponade, external cardiac restraint causes underfilling of the left ventricle, which leads to Decreased cardiac output. The amount of pericardial fluid required to become hemodynamically significant depends on the rate of accumulation and the compliance of the pericardium. Acute pericardial effusion can result in tamponade with as little as 200 mL of fluid orleven less in patients with thickened or scarred pericardium (ie, effusive-During inspiration, intrathoracic pressure falls, which leads to Increased filling of the right ventricle. Normally, the right ventricle can accommodate thise. constrictive pericarditis). Effusions over 1L in size may not be hemodynamical y significant if the rate of accumulation is slow. 12 Pulsus paradoxus is the decrease in systolic blood pressure of 10 mm Hg or greater during inspiration. It occurs as a result of ventricular interdependenc increase by expanding outward. However, in the setting of external cardiac restraint, outward expansion of the right ventricle becomes limited. Instead, the interventricular septum bows toward the left ventricle, impairing left ventricular filling and resulting in a decrease in cardiac output and blood pressure.12 Patients with cardiac tamponade and concomitant hypovolemia benefit from intravenous fluid administration. However, definitive treatment for cardiac tamponade is urgent evacuation of the pericardial space. 12
497 Key Points
Syncope is the transient loss of consciousness resulting from Syncope is a common problem in the general population, with a cerebral hypoxia, with spontaneous revival. variety of underlying causes that range from benign to life- threatening.
Syncope most often occurs as a result of Decreased cerebral blood cause syncope without compromised cerebral blood flow. categories: cardiovascular, neurocardiogenic, neurologic, and Cardiovascular syncope generally occurs as a result of Decreased flow. Anemia and hypoxemia are examples of conditions that can The causes of syncope can be separated into the following other. cardiac output. Neurocardiogenic syncope occurs as a result of either a neurally
mediated reflex that results in.bradycardia and vasodilation, orex impairment of the normal neurally mediated compensatory refl that maintains blood pressure
498
References 1. Saklani P, Krahn A, Klein G. Syncope. Circulation. 2013;127(12):1330-1339. 2. Mangrum JM, DiMarco JP. The evaluation and management of bradycardia. N Engl J Med. 2000;342(10):703-709.
-
Merideth J, Pruitt RD. Cardiac arrhythmias. 5. Disturbances in cardiac conduction and their management. Circulation. 1973;47(5):1098-1107.
-
Vogler J, Breithardt G, Eckardt L. Bradyarrhythmias and conduction blocks. Rev Esp Cardiol (Engl Ed). 2012;65(7):656-667.
-
Reynolds HR, Hochman JS. Cardiogenic shock: current concepts and improving outcomes. Circulation. 2008;117(5):686-697.
-
Ondrus T, Kanovsky J, Novotny T, Andrsova I, Spinar J, Kala P. Right ventricular myocardial infarction: from pathophysiology to prognosis. Exp Clin Cardiol.
2013;18(1):27-30. 7. Torbicki A, Perrier A, Konstantinides S, et al. Guidelines on the diagnosis and management of acute pulmonary embolism: the Task Force for the Diagnosis and
Management of Acute Pulmonary Embolism of the European Society of Cardiology (ESC). Eur Heart J. 2008;29(18):2276-2315.
-
Ross J Jr, Braunwald E. Aortic stenosis. Circulation. 1968;38(1 suppl):61-67. 9. Nishimura RA, Holmes DR Jr. Clinical practice. Hypertrophic obstructive cardiomyopathy. N Engl J Med. 2004;350(13):1320-1327.
-
Criado FJ. Aortic dissection: a 250-year perspective. Tex Heart Inst J. 2011;38(6):694- 700.
-
Yock PG,lPopp RL. Noninvasiveiestimation of right ventricular systolic pressure by Doppler u trasound in patients w th tricuspid regurgitation. Circulation. 1984;70(4):657-662.
-
Spodick DH. The.Pericardium: A Comprehensive Textbook. New York, NY: Marcel Dekker, Inc.; 1997
-
Potter BJ, Pinto DS. Subclavian steal syndrome. Circulation. 2014;129(22):2320-2323. 14. Chen-Scarabelli C, Scarabelli TM. Neurocardiogenic syncope. BMJ. 2004;329(7461):336-341.
-
MathiastCJ, DeguchitK, Schatz I. Observations on recurrent syncope and presyncope in 641 pa ients. Lance . 2001;357(9253):348-353.
- Soteriades ES, Evans JC, Larson.MG, et al. Incidence and prognosis of syncope. N Engl J Med. 2002;347(12):878-885
-
Grubb BP. Neurocardiogenic syncope and related disorders of orthostatic intolerance. Circulation. 2005;111(22):2997-3006.
-
Thieben MJ, Sandroni P, Sletten DM, et al. Postural(orthostatic tachycardia syndrome: 19. Jia X, Kowalski RG, Sciubba DM, Geocadin RG. Critical care of traumatic spinal cord the Mayo clinic experience. Mayo Clin Proc. 2007;82 3):308-313. injury. J Intensive Care Med. 2013;28(1):12-23.
-
Sheldon R, Rose S, Ritchie D, et al. Historical criteria that distinguish syncope from seizures. J Am Coll Cardiol. 2002;40(1):142-148. . Cryer PE, Axelrod L, Grossman AB, et al. Evaluation and management of adult
21hypoglycemic disorders: an Endocrine Society Clinical Practice Guideline. J Clin Endocrinol Metab. 2009;94(3):709-728.
499
-
Davidson E, Rotenbeg Z, Fuchs J, Weinberger I,.Agmon J. Transient ischemic attack-23. Savitz SI, Caplan LR. Vertebrobasilar disease. N Engl J Med. 2005;352(25):2618-2626. 24patients older than 65 years admitted to emergency departments for transient loss of related syncope. Clin Cardiol. 1991;14(2):141-144 . Mussi C, Ungar A, Salvioli G, et al. Orthostatic hypotension as cause of syncope in consciousness. J Gerontol A Biol Sci Med Sci. 2009;64(7):801-806.
-
Sathyapalan T, Aye MM, Atkin SL. Postural hypotension. BMJ. 2011;342:d3128.
-
Marino PL. The ICU Book. 3rd ed. Philadelphia, PA: Lippincott Williams & Wilkins—a Wolters Kluwer business; 2007.
-
Raichle ME, Plum F. Hyperventilation and cerebral blood flow. Stroke. 1972;3(5):566- 575.
-
Raj V, Rowe AA, Fleisch SB, Paranjape SY, Arain AM, Nicolson SE. Psychogenic pseudosyncope: diagnosis and management. Auton Neurosci. 2014;184:66-72.